Maxillo-facial Trauma 01-22 Flashcards

1
Q

An 18-year-old man presents with right periorbital edema and ecchymosis after an all-terrain vehicle collision. Physical examination shows enophthalmos, diplopia, and pain with eye movements. When asked to look upward from forward gaze, there is upward gaze restriction. A photograph is shown. Which of the following locations is most likely fractured?

A) Greater wing of sphenoid bone
B) Lamina papyracea of ethmoid bone
C) Orbital plate of frontal bone
D) Orbital process of maxillary bone
E) Posterior crest of lacrimal bone

A

The correct response is Option D.

Limitation with vertical gaze, as described in the vignette, is indicative of extraocular muscle (EOM) entrapment.

The lamina papyracea of the ethmoid bone contributes to the medial orbital wall. Fractures of the medial wall may result in medial rectus entrapment and restriction with lateral gaze. Such injuries can involve the ethmoidal/sphenoidal sinuses.

The greater wing of the sphenoid bone contributes to the lateral orbital wall. Fractures of the lateral wall are less common due to its increased strength, but at the wings of the sphenoid, bone can have serious effects on contents of the superior orbital fissure and may even involve the intraorbital portion of the optic (II) nerve.

The orbital plate of the frontal bone contributes to the superior orbital roof. Fractures of the superior roof may result in superior rectus and/or oblique entrapment. Such injuries can involve the frontal sinus, frontal lobe, supraorbital nerve, and/or supratrochlear nerve (resulting in loss of sensation of the forehead and upper eyelid).

The orbital process of the maxillary bone contributes to the inferior orbital floor. Fractures of the inferior floor may result in inferior rectus/oblique entrapment and restriction with upward gaze. Such injuries can involve the maxillary sinus and/or infraorbital nerve (resulting in malar and superior alveolar numbness).

The posterior crest of the lacrimal bone contributes to the medial orbital wall. Fractures of the medial wall may result in medial rectus entrapment and restriction with lateral gaze. Such injuries can involve the ethmoidal/sphenoidal sinuses.

How well did you know this?
1
Not at all
2
3
4
5
Perfectly
2
Q

An otherwise healthy 74-year-old man presents with traumatic brain injury from an open comminuted anterior and posterior table frontal sinus fracture sustained in a motor vehicle collision. Persistent clear fluid rhinorrhea is observed when the patient is upright. The patient is initially medically unstable, and surgical intervention is delayed by 5 days. Which of the following factors represents the greatest increase in risk for a central nervous system infection?

A) Cerebrospinal fluid leak
B) More than 48 hours before operative repair
C) Open fracture injury
D) Patient age
E) Presence of traumatic brain injury

A

The correct response is Option B.

There are many algorithms for treating frontal sinus fractures. Unfortunately, many of these patients have other injuries that may limit their surgical options. Conservative management has been shown to be successful at managing comminuted, displaced fractures, pre-injury comorbidities, or those with cerebrospinal fluid (CSF) leak. Factors that are not associated with an increased risk for serious infection include: preoperative CSF leak, persistent CSF leak, surgical procedure performed, age, gender, and penetrating or open injury. Factors that do impact the risk for serious infection include: greater than 48 hours from injury to operating room intervention, use of CSF catheter diversion, and soft-tissue infection.

How well did you know this?
1
Not at all
2
3
4
5
Perfectly
3
Q

A 20-year-old man has severe hypotension and bradycardia after sustaining multiple facial fractures in a motor vehicle collision. He has no other injuries. Repairing which of the following structures will most likely improve the patient’s symptoms?

A) Mandible
B) Mental nerve
C) Nasal bone
D) Orbital floor
E) Zygomatic arch

A

The correct response is Option D.

The oculocardiac reflex can precipitate marked bradycardia and hypotension in the setting of trauma with significant orbital and maxillofacial injury. Prompt identification and management with vagolytic agents or definitive surgical intervention may prevent morbidity or mortality. Patients who sustain maxillofacial trauma involving the orbit, most commonly the orbital floor, are at risk for developing the oculocardiac reflex. These patients tend to be young, and common symptoms include nausea and vomiting. The oculocardiac reflex is not static and may evolve during a patient’s clinical course.

How well did you know this?
1
Not at all
2
3
4
5
Perfectly
4
Q

Elderly patients are more likely than younger patients to have which of the following facial fractures?

A) Nonoperative Le Fort fracture from a bicycle accident
B) Nonoperative mandible fracture from a motor vehicle collision
C) Nonoperative maxillary fracture from a fall
D) Operative nasal fracture from interpersonal violence
E) Operative orbital fracture from a workplace injury

A

The correct response is Option C.

Elderly patients are more likely to sustain injury from falls than younger patients and are less likely to be in fights. Nonoperative maxillary fractures and nasal bone fractures are more common in elderly patients, and mandible fractures are more common in younger patients.

How well did you know this?
1
Not at all
2
3
4
5
Perfectly
5
Q

A 50-year-old man receives preoperative radiation therapy for a large calvarial osteosarcoma that will require a 9-cm2 craniectomy. Which of the following is the most appropriate material to use for reconstruction?

A) Bone allograft with bone morphogenic protein (BMP)
B) Calcium phosphate
C) Polyether ether ketone (PEEK) implant
D) Titanium mesh

A

The correct response is Option C.

Any calvarial defect greater than 6 cm2 should be reconstructed. Titanium mesh can be used for craniectomy reconstruction; however, radiologic surveillance of disease recurrence can be challenging with the degree of artifact seen on imaging. Polyether ether ketone (PEEK) implants are radiolucent and therefore do not produce artifact on surveillance imaging and can be customized based on the size of the planned resection. Bone allograft with bone morphogenic protein (BMP) is contraindicated in cases of malignancy since BMP is a growth factor that induces bone formation and is involved in tumorigenesis. Calcium phosphate can be used for smaller calvarial defects, but it is associated with a high risk for infection and would be contraindicated in a patient who received preoperative radiation therapy.

How well did you know this?
1
Not at all
2
3
4
5
Perfectly
6
Q

A 50-year-old man sustained multiple visceral injuries, prolonged loss of consciousness, and a fracture of the orbital floor in an accident 3 months ago. He deferred repair of the orbital floor fracture at the time of injury, but is now seeking help for symptoms related to the fracture. The patient is alert and oriented. Orbital floor fracture repair is most likely to achieve correction of which of the following findings in this patient?

A) Blindness
B) Ectropion
C) Enophthalmos
D) Globe volume
E) Vertical restriction

A

The correct response is Option C.

Orbital floor fractures risk the pathoanatomy of changing the volume of the orbit, which can affect the position of the globe, as well as entrapping the inferior rectus muscle, leading to restriction of globe movement. Therefore, indications for surgery include: vertical globe dystopia (vertical change in globe position from inferior displacement of floor), enophthalmos (retro-positioning of globe from increased orbit volume), and globe entrapment (inability for globe to look up).

For this patient 3 months after unrepaired orbital floor fracture, enophthalmos would be the most likely correctable by delayed orbital floor repair, which reduces the orbital volume closer to its pre-injury state, thereby restoring the globe to its pre-injury position.

Blindness would be from optic nerve injury, which would not be improved by orbital floor fracture repair.

Ectropion would usually be from lower eyelid contracture, which would not be improved by orbital floor fracture repair.

Globe volume refers to volume of the globe itself (not the orbit volume), which would not be improved by orbital floor fracture repair.

Vertical restriction could be due to muscle entrapment, which needs to be repaired urgently otherwise permanent damage to the muscle could result. This patient is presenting 3 months after injury for delayed repair, therefore orbital floor fracture repair would no longer be able to repair a permanently damaged muscle entrapment scenario.

How well did you know this?
1
Not at all
2
3
4
5
Perfectly
7
Q

A 30-year-old woman presents with the dental findings shown in the diagram. Which of the following best describes the dental relationship?

A) Angle class I
B) Angle class II
C) Angle class III
D) Negative overjet
E) Overbite

A

The correct response is Option B.

The images show an Angle class II relationship. The Angle classification system describes the relative positions between the mesial buccal cusp of the maxillary first molar and the buccal groove of the mandibular first molar. Angle class I molar relationship implies that the mesiobuccal cusp is in line with the buccal groove. In an Angle class II molar relationship, the maxillary mesiobuccal cusp is anterior to the mandibular buccal groove. Class II is subdivided into two divisions. In class II, division 1, patients have minimal crowding of the maxillary teeth and proclination of the upper central incisors, and a significantly increased overjet. In a class II, division 2 relationship, the central incisors are retroclined. An Angle class III molar relationship exists when the maxillary mesiobuccal cusp lies posterior to the mandibular buccal groove.

How well did you know this?
1
Not at all
2
3
4
5
Perfectly
8
Q

A 23-year-old man sustains a severe right orbital floor fracture in a physical altercation. Reconstruction with a pre-bent orbital floor plate and intraoperative CT scanning is planned. Which of the following is the most likely to be optimized using this imaging modality intraoperatively?

A) Operative time
B) Plate positioning
C) Rate of corneal abrasion
D) Rate of plate extrusion
E) Risk for lid malposition

A

The correct response is Option B.

Use of intraoperative computed tomography has been gaining traction in maxillofacial trauma. Stated benefits include decreased re-operation rate, improved accuracy of plate positioning, and decreased postoperative enophthalmos. Although the use of intraoperative CT scans may increase time in the operating room, it has no effect on rates of corneal abrasion, lid malposition, or plate extrusion.

How well did you know this?
1
Not at all
2
3
4
5
Perfectly
9
Q

Which of the following best represents the likelihood that a patient with a frontal sinus fracture would have a concurrent intracranial injury?

A) 1%
B) 15%
C) 30%
D) 55%
E) 90%

A

The correct response is Option D.

In an acute trauma setting, the recognition of mild traumatic brain injury (mTBI) is a diagnostic challenge as there are often competing diagnoses that take immediate priority. Furthermore, within this cohort, patients with craniofacial fractures have been shown to be at risk for delayed or missed diagnosis for all degrees of TBI, although with a higher likelihood of missed or delayed diagnosis for mTBI compared with moderate to severe TBI. Previously, it was hypothesized that facial fractures buffered the forces transmitted during blunt head trauma, thereby protecting intracranial structures. This conceptual framework has since been questioned as evidence has mounted that individuals with facial fractures are at increased risk for head injury. The biomechanics resulting in different types of facial fractures and the amount of force required to fracture the different components of the facial bony structure have been well described. The nasal bone has the lowest tolerance for fracture at 25 to 75 lbs, while the frontal bone has the highest tolerance at 800 to 1600 lbs. Recent studies have proposed that craniofacial fractures can serve as clinical markers for brain injury and Mulligan et al. suggest that the prevalence of overall head and cervical spine injuries in the setting of facial fractures is high enough to warrant a change in current protocols.

In this context, the prevalence of mTBI and moderate to severe TBI in patients with isolated facial fractures in the National Trauma Databank (NTDB) was evaluated, and further characterized the association of isolated facial fractures with different degrees of TBI in patients with mild, moderate, and severe TBI. Facial fractures can serve as objective clinical markers for the potential presence of mTBI and moderate to severe TBI in trauma patients. As mTBI patients have been shown to benefit from simple, easy-to-administer educational interventions, trauma patients with facial fractures may benefit from automatically receiving education about mTBI and TBI recovery, given the clinically meaningful prevalence of mTBI and TBI in this population. As one moves up the craniofacial skeleton, the forces are transmitted more reliably to the intracranial space. Therefore, a frontal sinus fracture is at extremely high risk (usually a 45 to 65% chance) of having an associated intracranial injury.

How well did you know this?
1
Not at all
2
3
4
5
Perfectly
10
Q

A 27-year-old man sustained multiple facial fractures when he was involved in a motorcycle collision. On arrival to the emergency department, blood pressure is 80/50 mmHg and heart rate is 150 bpm. Significant retropharyngeal bleeding is noted. Trauma workup reveals no other injuries. CT angiography shows active bleeding from the right maxillary artery. Angioembolization is planned and massive transfusion protocol is initiated. Which of the following is the most appropriate intravenous resuscitation in this patient?

A) Fresh frozen plasma (FFP) and packed red blood cells (pRBC) in a 1:1 ratio; discontinuation of crystalloids
B) FFP and pRBC in a 1:1 ratio; crystalloids via rapid transfuser (max rate)
C) FFP and pRBC in a 1:4 ratio; crystalloids at 125 cc/h
D) FFP and pRBC in a 1:4 ratio; discontinuation of crystalloids
E) FFP and pRBC in a 4:1 ratio; crystalloids via rapid transfuser (max rate)

A

The correct response is Option A.

For initiation of a massive transfusion protocol, transfusing fresh frozen plasma (FFP) and packed red blood cells (pRBC) at a 1:1 ratio and discontinuing intravenous crystalloids is the most appropriate next step in patient management.

Massive Transfusion Protocol guidelines have been set forth by the American College of Surgeons through its Trauma Quality Improvement Program (TQIP). Recommendations for initiating a massive transfusion protocol include:

Beginning universal blood product infusion rather than crystalloid or colloid solutions,

Transfusing universal pRBC and FFP in a ratio between 1:1 and 1:2 (FFP:pRBC),

Transfusing one single donor apheresis or random donor platelet pool for each six units of pRBC.

It is also suggested to deliver PRBC and FFP by a rapid transfuser and through a blood warmer, and that the initial rate of transfusion should restore perfusion while allowing for “permissive hypotension” until the operation or angioembolization to stop the bleeding begins.

How well did you know this?
1
Not at all
2
3
4
5
Perfectly
11
Q

A 65-year-old man develops a hemorrhagic stroke requiring decompressive craniotomy. The bone is found to be unusable and a customized polyetheretherketone prosthesis is planned. Which of the following is the most common complication of using this material?

A) Cerebrospinal fluid leak
B) Contour deformity
C) Dehiscence
D) Hematoma
E) Infection

A

The correct response is Option E.

Reports on using polyetheretherketone (PEEK) as an alloplast for cranial reconstruction vary in terms of outcomes and complications. The larger studies conclude that it is a reliable material compared with other alloplastic alternatives and has the advantage of being custom made for a variety of craniofacial defects. However, infection remains the most common complication, and choosing this material should be weighed against the risk for microorganism seeding through, wound dehiscence, hematogenous spread, or indolent colonization of the wound bed.

How well did you know this?
1
Not at all
2
3
4
5
Perfectly
12
Q

A 32-year-old man comes to the emergency department after being hit in the right eye. Examination shows enophthalmos, hyphema, and numbness over the cheek. There is no diplopia. CT scan shows a large orbital floor fracture with herniation of contents into the maxillary sinus. Which of the following findings requires urgent management?

A) Cheek numbness
B) Enophthalmos
C) Hyphema
D) Maxillary sinusitis
E) Orbital floor fracture

A

The correct response is Option C.

Hyphema is marked by presence of blood in the anterior chamber and is an emergent concern. It can lead to permanent damage to the vision. All the other options are urgent concerns but can be addressed after the hyphema is treated.

How well did you know this?
1
Not at all
2
3
4
5
Perfectly
13
Q

A 65-year-old man who wears glasses sustained a massive injury to the left side of the face causing a ruptured globe with total loss of the upper and lower eyelids. Which of the following is the best aesthetic option to recommend?

A) Eye patch
B) Hemifacial prosthesis
C) Ocular prosthesis
D) Orbital prosthesis

A

The correct response is Option D.

In this case the patient has had severe orbital trauma with loss of lids and globe. Natural-looking and functional total-lid reconstruction is challenging. Lids would be needed to support an ocular prosthesis. An orbital prosthesis would likely provide this patient a comfortable and aesthetically satisfactory prosthesis. Eyeglasses can help mask the seam of the prosthesis. The hemifacial prosthesis is larger than necessary for this patient and has unnatural seams. An eye patch would not improve symmetry or be reconstructive.

How well did you know this?
1
Not at all
2
3
4
5
Perfectly
14
Q

In a patient undergoing reconstructive cranioplasty, an increased rate of complications is most likely if which of the following is present?

A) Frontal location
B) Occipital location
C) Parietal location
D) Sphenoidal location
E) Temporal location

A

The correct response is Option A.

Early decompressive craniectomy is a life-saving maneuver for certain traumatic brain injuries and can be performed far forward in the theater of war. Patients treated with decompressive craniectomy for combat injuries are a unique understudied population. Outcome of treatment of this patient cohort has been previously reported using a standardized cranial defect treatment protocol using custom alloplast implants. Two subgroups of patients (large endocranial dead space and frontal orbital bar injuries) were identified as often having higher rates of complications than other cranial reconstruction cohorts.

How well did you know this?
1
Not at all
2
3
4
5
Perfectly
15
Q

A 28-year-old man is brought to the emergency department after sustaining injury during a motor vehicle collision. Cranialization of the frontal sinus is planned. Which of the following best describes the components of cranialization?

A) Removal of the anterior table, reconstruction of the posterior table with a titanium plate, and closure of the dura
B) Removal of the posterior table, sinus mucosa, and closure of the sinonasal tract
C) Repair of both the posterior and anterior tables with bioabsorbable plates, and obliteration of the frontal sinus
D) Repair of the anterior table and obliteration of the frontal sinus
E) Repair of the posterior table with bioabsorbable plates, removal of the sinus mucosa, and closure of the dura

A

The correct response is Option B.

Cranialization involves removal of the posterior table (not repair), closure of the dura, sinonasal tract, and obliteration of the sinus mucosa. Management of the anterior table is as indicated.

Surgical repair of the anterior table is indicated if there is nasofrontal duct involvement, or, in the absence of nasofrontal duct involvement (such as a minimally displaced anterior table), patient desire for a better aesthetic outcome. If there is nasofrontal duct involvement, the nasofrontal duct and frontal sinus can be obliterated (repair of the anterior table and obliteration of the frontal sinus).

Bioabsorbable or titanium plates can be used to fixate the fractured anterior table. It is not used for the posterior table.

How well did you know this?
1
Not at all
2
3
4
5
Perfectly
16
Q

A 30-year-old man sustains significant mid face injuries following a motor vehicle collision, and has a large laceration in the vicinity of the medial canthal region. Canalicular injury is confirmed intra-operatively. Which of the following is the most appropriate method for repairing this patient’s canalicular injury?

A) Delayed dacryocystorhinostomy
B) Direct microsurgical suture repair
C) Healing by secondary intention
D) Immediate dacryocystorhinostomy
E) Placement of silicone canalicular stents

A

The correct response is Option E.

When canalicular injury is suspected, the lacrimal system should be investigated for patency. Typically, this involves performing a Jones I and II test to determine if fluorescein navigates from the lower lid fornix into the nose. If canalicular interruption is suspected and identified, the proximal and distal stumps of the canaliculus are joined by placing a silicone stent and leaving this in place for 3 to 6 months to allow for healing.

Direct microsurgical suturing is not preferred because of the high likelihood of cicatricial obstruction.

Dacryocystorhinostomy is generally reserved as a “salvage” procedure for patients who have lacrimal obstruction after being treated with a stent. Healing by secondary intention is incorrect since it would likely result in canalicular obstruction.

How well did you know this?
1
Not at all
2
3
4
5
Perfectly
17
Q

A 40-year-old man and his 80-year-old father are assaulted. They both have facial fractures. The older victim is more likely to have which of the following?

A) Decreased chance of noncraniofacial injuries
B) Higher mortality
C) Less severe injuries
D) Mandibular body fracture
E) Shorter hospital stay

A

The correct response is Option B.

In recent years many publications focused on craniofacial injury in the elderly as not only the mode of trauma differs compared with the younger population, but also the associated injuries and morbidities. In general, most related comorbidities in patients older than 60 to 65 (depending on the study) versus those younger are worse, including: longer hospital stays, need for assistance upon discharge, more severe injuries, likely to have noncraniofacial injuries like limb and spine fractures, and, of greatest concern, a much higher death rate. In a recent article though, Mundinger et al, showed that panfacial and mandible fractures were more common in the nongeriatric population, whereas mid face, orbital, and condylar fractures were more common in those older than 60 years of age.

How well did you know this?
1
Not at all
2
3
4
5
Perfectly
18
Q

A 20-year-old man desires correction of a depressed, retracted, post-tracheostomy scar. Which of the following is the best recommendation for improving the scar?

A) Perform autologous fat grafting and laser resurfacing
B) Reconstruction tracheal ring and detach adhesions
C) Scar excision and interposition of acellular dermal matrix
D) Scar excision and reapproximation of strap muscles
E) Scar revision

A

The correct response is Option D.

After decannulation, the tracheostomy site heals by secondary intention. Often the patient is left with a soft, small asymptomatic scar. On occasion, the scar is painful and the skin has adhesions to tissue deep to the strap muscles. This may lead to pulling and retraction with swallowing as well as a scar that is not aesthetically pleasing to the patient. The depressed retracted tracheostomy scar requires reapproximation of platysma and approximation of the sternothyroid and sternohyoid for correction. Fat grafting is unlikely to address retraction or fully correct the depression. Laser resurfacing and fat grafting will have minimal improvement of retraction. Several studies support use of cadaver materials or fascia to support the coverage of the strap muscles when tissue is missing or heavily damaged. The tracheal ring does not need to be reconstructed for routine tracheostomy scar revision. Care must be taken when working around the trachea. Communication with anesthesia about oxygen content and fire risk is important for surgical safety.

How well did you know this?
1
Not at all
2
3
4
5
Perfectly
19
Q

A patient underwent open reduction and internal fixation of naso-orbital-ethmoid fractures 12 months ago, and epiphora was noted on follow-up examination. After 6 months of observation and persistent epiphora, which of the following is the most appropriate next step to evaluate the function of the patient’s nasolacrimal system?

A) Conjunctivorhinostomy tube placement
B) Continued observation, as function is likely to return
C) Jones tests
D) Lacrimal system flushing
E) Schirmer tests

A

The correct response is Option C.

The Jones test is used to evaluate lacrimal drainage. Divided into two parts, the Jones I test investigates lacrimal outflow under normal physiologic conditions. A drop of sterile 2% fluorescein solution or a moistened fluorescein strip is placed into the conjunctival fornix and a cotton-tipped wire applicator is placed into the inferior nasal meatus in the region of the ostium of the nasolacrimal duct at 2 and 5 minutes to check for fluorescein. As this test occasionally yields abnormal results in normal patients, it is not uniformly performed. The Jones II test determines the presence or absence of fluorescein when the residual fluorescein is flushed from the conjunctival sac with clear saline to determine whether there is reflux of fluorescein.

Naso-orbital-ethmoid (NOE) fractures can be challenging fractures, and either through direct instrumentation with transcanthal wiring or from the fractures themselves, the lacrimal drainage system can be affected. Postoperative epiphora can be very common and is present in at least 50% of patients who have undergone open reduction and internal fixation (ORIF) of an NOE fracture. After 3 to 6 months approximately half of this epiphora resolves, with the other half of patients (25%) requiring consideration for other investigations to evaluate lacrimal drainage. Schirmer test is used to evaluate dry eyes and is not appropriate in this patient.

How well did you know this?
1
Not at all
2
3
4
5
Perfectly
20
Q

A 22-year-old man is evaluated for multiple facial fractures after he was assaulted. Which of the following fractures is most likely associated with an increased risk of temporomandibular joint dysfunction?

A) Bilateral parasymphyseal mandible
B) Comminuted unilateral condylar mandible
C) Complete Le Fort I maxillary
D) Displaced unilateral subcondylar mandible
E) Unilateral zygomaticomaxillary

A

The correct response is Option B.

Temporomandibular joint (TMJ) dysfunction symptoms are serious, often overlooked complications of facial fractures and their treatments. They can range from clicking and pain to locking, malocclusion, and trismus. Overt ankylosis can occur in rare circumstances. Fractures that result in significant disruption of the condylar/glenoid apparatus are more likely to result in TMJ dysfunction symptoms than more anatomically remote fractures. Condylar fractures are most susceptible to post-fracture TMJ dysfunction. This is especially true in comminuted condylar head fractures. One recent study demonstrated an increase in TMJ dysfunction symptoms in patients with condylar fractures and concomitant contralateral mandibular body/angle fractures. Le Fort I and zygomaticomaxillary complex (ZMC) fractures are unlikely to be associated with TMJ symptoms.

How well did you know this?
1
Not at all
2
3
4
5
Perfectly
21
Q

A 7-year-old boy is brought to the emergency department after being injured in a domestic violence incident. Physical examination shows bruising around the right eye. The patient reports pain and nausea when looking upward. A CT scan shows an entrapped inferior rectus muscle. Three weeks later, the floor of the orbit is repaired with an orbital floor implant. One year later, he continues to have diplopia. Which of the following is the most likely reason for the persistent diplopia?

A) Exophthalmos
B) Location of prosthesis
C) Nerve damage
D) Persistent swelling
E) Timing of surgery

A

The correct response is Option E.

Timing of pediatric orbital floor fractures is well studied. Unlike adult fractures, significant delays for surgery in children, especially more than 7 days after injury, is associated with varying degrees of diplopia. Many consider this pathology an emergency and should be treated within 24 hours. Assuming a typical, standard of care approach is performed well from a technical standpoint, only delays in time to treat were shown to predict such a poor outcome.

How well did you know this?
1
Not at all
2
3
4
5
Perfectly
22
Q

Proper reduction of a zygomaxillary complex (ZMC) fracture requires reduction and realignment of which of the following?

A) Zygomatic arch, infraorbital rim, alveolus
B) Zygomaticofrontal suture, infraorbital rim, alveolus
C) Zygomaticofrontal suture, zygomaticomaxillary buttress, infraorbital rim
D) Zygomaticofrontal suture, zygomaticomaxillary buttress, orbital floor
E) Zygomaticomaxillary buttress, infraorbital rim, alveolus

A

The correct response is Option C.

A zygoma fracture involves displacement of the zygoma that articulates with the frontal bone, maxilla, and sphenoid. In order to stabilize the fracture after adequate reduction, the zygomaticofrontal, zygomaticomaxillary buttress, and infraorbital rim need to be fixated. If there is a large (>2 cm2) defect in the orbital floor after reduction, reconstruction of the orbital floor is also necessary to prevent enophthalmos.

While the nasomaxillary buttress is one of the vertical buttresses of the face, the zygoma does not articulate with the nasal bones.

How well did you know this?
1
Not at all
2
3
4
5
Perfectly
23
Q

A 10-year-old boy is brought to the physician after sustaining a nondisplaced fracture of the mandibular body in a fall. Soft diet is recommended. Two days later, he is brought back to the office and reports pain in the right mandibular lateral incisor when drinking cold liquid. The base of the defect appears yellow and is tender when probed. Examination shows a lingual fracture of the tooth crown. On the basis of these findings, which of the following is the deepest layer of exposed tooth?

A) Cementum
B) Dentin
C) Enamel
D) Pulp cavity
E) Root canal

A

The correct response is Option B.

This patient has a fracture of the tooth crown that extends through the dental enamel into the deeper parts of the tooth. This is evidenced by the sensitivity to touch and cold, a finding not characteristic of a fracture limited to the enamel. The yellow color to the base of the fracture indicates exposed dentin, which resides just under the hard outer enamel layer of the tooth. If the fracture had extended deeper into the pulp cavity, the area where the vessels and nerves reside, the base of the fracture would appear as a blood-filled cavity. These injuries often challenge the viability of the tooth and often require drilling and packing of the pulp space (root canal). The fracture described is of the crown and there is no indication that it involves the root of the tooth or the surrounding structures. Cementum is a bone-like covering of the tooth root and would not be affected by this injury.

The Ellis classification provides a useful system of categorizing these injuries. There are 9 categories:

Ellis I: enamel fracture. The tooth is non tender and treatment is smoothing of the rough surfaces and, possibly, application of a filling or amalgam.
Ellis II: fracture of the enamel and dentin. Tooth is tender to air, cold, and probing and the base of the defect often appears yellow.
Ellis III: involves the enamel, dentin, and the pulp space. The tooth is sensitive as in Ellis II, but the base of the defect appears red or bloody.
Ellis IV: a nonviable tooth.
Ellis V: luxation of the tooth.
Ellis VI: tooth avulsion.
Ellis VII: displacement without fracture.
Ellis VIII: fracture of entire crown.
Ellis IX: fracture of deciduous teeth.

How well did you know this?
1
Not at all
2
3
4
5
Perfectly
24
Q

Which of the following is the most common complication of a fracture to the temporal bone?

A) Cerebrospinal fluid leak
B) Facial nerve injury
C) Hearing loss
D) Meningitis
E) Temporomandibular joint ankylosis

A

The correct response is Option A.

Cerebrospinal fluid leak is the most common complication of a temporal bone injury. The majority of these will resolve spontaneously within a week. If they persist longer, then there is higher risk for meningitis, but this is not common. Facial nerve injury is the second most common injury and prognosis is dependent on the severity and delay of onset. Incomplete nerve loss or delayed onset is associated with a better prognosis for recovery. Hearing loss is the third most common complication seen with this fracture. Temporomandibular joint ankylosis is an unlikely sequela of this type of injury.

How well did you know this?
1
Not at all
2
3
4
5
Perfectly
25
Q

In adults, which of the following bones is most commonly fractured in isolated orbital floor fractures?

A) Ethmoid
B) Frontal
C) Lacrimal
D) Maxillary
E) Zygomatic

A

The correct response is Option D.

Most isolated orbital fractures involve the orbital floor made up of the maxillary bone. The maxillary bone is quite thin behind the infraorbital rim and is perforated by the infraorbital nerve passing in a canal below it. Most pure blow-out fractures involve the orbital floor with the maxillary bone making the majority of the orbital floor.

A retrospective study by Hwang et al. evaluated 391 patients with orbital bone fractures from a variety of accidents that were treated at the department of Plastic and Reconstructive Surgery, Inha University Hospital, Incheon, South Korea, between February 1996 and April 2008. The medical records of these patients were reviewed and analyzed to determine the clinical characteristics and treatment of the orbital bone fractures. The following results were obtained. The mean age of the patients was 31.1 years, and the age range was 4 to 78 years. The most common age group was the third decade of life (32.5%). There was a significant male predominance in all age groups, with a ratio of 4.43:1. The most common etiology was violent (assault) or nonviolent traumatic injury (57.5%) followed by traffic accidents (15.6%) and sports injuries (10.7%). The most common isolated orbital bone fracture site was the orbital floor (26.9%). The largest group of complex fractures included the inferior region of the orbital floor and zygomaticomaxilla (18.9%). Open reduction was performed in 63.2% of the cases, and the most common fracture reconstruction material was MEDPOR (56.4%) followed by a resorbable sheet (41.1%). The postoperative complication rate was 17.9%, and there were no statistically significant differences among the reconstruction materials with regard to complications. During follow-up, diplopia, hypoesthesia, and enophthalmos occurred as complications; however, there was no significant difference between porous polyethylene sheet (MEDPOR) and resorbable sheet groups.

Long-term epidemiologic data regarding the natural history of orbital bone fractures are important for the evaluation of existing preventative measures and for the development of new methods of injury prevention and treatment.

How well did you know this?
1
Not at all
2
3
4
5
Perfectly
26
Q

A patient underwent open reduction and internal fixation of naso-orbital-ethmoid fractures 12 months ago, and epiphora was noted on follow-up examination. After 6 months of observation and persistent epiphora, which of the following is the most appropriate next step in management of the patient’s nasolacrimal system?

A) Conjunctivorhinostomy tube placement
B) Continued observation
C) Dacryocystorhinostomy
D) Jones tests
E) Lacrimal system flushing

A

The correct response is Option D.

Naso-orbital-ethmoid (NOE) fractures can be challenging fractures, and either through direct instrumentation with transcanthal wiring or from the fractures themselves, the lacrimal drainage system can be affected. Postoperative epiphora can be very common and is present in at least 50% of patients who have undergone open reduction and internal fixation (ORIF) of an NOE fracture. After 3 to 6 months approximately half of this epiphora resolves, with the other half of patients (25%) requiring consideration for other investigations to evaluate lacrimal drainage.

How well did you know this?
1
Not at all
2
3
4
5
Perfectly
27
Q

A 22-year-old man comes to the office because of a history of nasal trauma with resultant nasal deformity, C-shaped septal fracture, and nasal obstruction. Two weeks after injury, he undergoes closed reduction of the nasal fractures, but significant nasal obstruction persists. Which of the following is the most likely reason for his residual nasal deformity and nasal obstruction?

A) Inadequate time of nasal casting
B) Nonunion of the nasal bones
C) Presence of a septal fracture
D) Turbinate hypertrophy
E) Unidentified septal hematoma

A

The correct response is Option C.

One of the most important causes of failure of closed reduction to address the nasal fracture is simultaneous nasoseptal fracture. Murray, et al. found 30 to 40% residual nasal deformity after closed reduction. The cadaver study showed failure consistently associated with a C-shaped nasoseptal deviation and fracture when the external nose deviated at least 1/2 of the nasal bridge width. The theory is that the interlocking of the septal fracture creates tension causing the nasal bone to displace.

Untreated septal hematoma results in thickening of the cartilage and nasal obstruction, but not with inadequate reduction. Nasal casting for 7 to 10 days is sufficient to allow the reduction to set. Nonunion is a rare cause of inadequate reduction, usually in comminuted or open nasal fractures. Turbinate hypertrophy can cause nasal obstruction but does not interfere with nasal bone reduction.

How well did you know this?
1
Not at all
2
3
4
5
Perfectly
28
Q

A 26-year-old woman who is at 32 weeks’ gestation sustains a traumatic head injury during a boating collision. CT scan shows subarachnoid hemorrhage and pan-facial fractures. The patient is cleared by the neurosurgeon for facial fracture repair. In the ICU, blood pressure is 112/70 mmHg and heart rate is 95 bpm. Fetal monitoring shows no distress. The patient is taken to the operating room and placed in supine position. On the operating table, blood pressure is 80/50 mmHg and heart rate is 130 bpm. Which of the following is the most appropriate next step in management?

A) Administer fluid bolus intravenously
B) Logroll the patient to the left
C) Obtain immediate chest x-ray study
D) Prepare and drape the patient for the planned procedure
E) Start vasopressors

A

The correct response is Option B.

The most appropriate next step in this scenario is to logroll the patient 4 to 6 inches (or 15 degrees) to the left, decompressing the inferior vena cava (IVC). Women in the second half of pregnancy may become hypotensive when placed in the supine position, caused by compression of the inferior vena cava by the enlarged uterus, reducing venous return to the heart by up to 30%. Spinal precautions should be maintained for any patient whose spine has not been appropriately cleared.

Vasopressors should be used as a last resort in restoring the blood pressure of pregnant trauma patients, as these drugs further reduce uterine blood flow, resulting in fetal hypoxia. The placental vasculature is exquisitely sensitive to catecholamine stimulation.

Crystalloid fluid resuscitation would be indicated if the patient’s vital signs did not return to baseline after repositioning and IVC decompression. Similarly, a chest X-ray could be obtained as part of the workup for unresponsive hypotension.

Ignoring this patient’s hemodynamic changes and proceeding with surgery would be a mistake, as the placenta would most likely be deprived of vital perfusion, resulting in fetal distress.

How well did you know this?
1
Not at all
2
3
4
5
Perfectly
29
Q

A 25-year-old man is brought to the emergency department after sustaining injury during a roll-over motor vehicle collision. CT scan shows multiple facial fractures and systemic injuries. Which of the following is a CONTRAINDICATION to nasotracheal intubation in a trauma patient?

A) Base of skull fracture
B) Bilateral mandibular condyle fractures
C) Cervical spine fracture
D) Depressed mental status
E) Fracture of the maxilla

A

The correct response is Option A.

Of the options listed, base of skull fracture is the only contraindication to nasal intubation. In these patients, there is a risk of the nasotracheal tube passing through the cribriform plate into the frontal lobes of the brain. Other absolute contraindications include mid face instability, suspected epiglottitis, coagulopathy, or apnea/impending respiratory arrest. Relative contraindications to nasotracheal intubation include nasal polyps, suspected nasal foreign bodies, recent nasal surgery, upper-neck hematoma or infection, and a history of frequent epistaxis. Depressed mental status, suspected cervical spine fracture, hypotension, and bilateral mandibular condyle fractures are not contraindications to nasotracheal intubation.

The options for intubating a trauma patient include nasal, oral, and surgical airways. The indications for nasotracheal intubation include intraoral and oropharyngeal surgery, complex intraoral procedures involving the mandible (e.g., segmental mandibulectomy, osteotomy, mandibular reconstructive procedures), and dental surgery.

The advantages of nasotracheal intubation include uninhibited access to the mouth as well as an enlarged surgical field. Despite these advantages, one must be cognizant of the contraindications and when in doubt (e.g., during initial advanced trauma life support management), the airway should be secured using either oral intubation or tracheostomy. Other drawbacks to nasotracheal intubation are that it often requires multiple attempts, there can be an abrupt rise in intracranial pressure, and it is difficult to attempt in an uncooperative patient.

In the presence of an isolated maxillary fracture nasal intubation may be preferred so that premorbid occlusion can be established.

The method of insertion of a nasotracheal tube includes using a well-lubricated tube with the cuff fully deflated. The tube should be inserted into either naris at a right angle to the face. Once the tube is beyond the nasopharynx, a laryngoscope is placed in the oral cavity and the tube is advanced under direct vision.

How well did you know this?
1
Not at all
2
3
4
5
Perfectly
30
Q

A 21-year-old man undergoes repair of the parotid gland and Stensen duct after sustaining an injury while being assaulted with a glass bottle. Ten days later, salivary leakage from the wound is noted. Which of the following is the most appropriate next step in management?

A) Pressure dressings and limited oral intake
B) Radiation therapy
C) Re-exploration of the wound and repair of the duct injury
D) Tympanic neurectomy
E) Observation only

A

The correct response is Option A.

Sialocele and cutaneous fistula may occur as a complication of a major salivary gland injury or after attempted parotid gland or duct repair. Prior to intervention, the surgeon should confirm the presence of salivary fluid in the aspirate by evaluating for amylase (>1000 U/L). This helps to rule out hematoma vs. infection. At this time, the accepted initial treatment of sialocele and cutaneous fistula is conservative management. This includes pressure dressings, repeated aspirations, limited intake by mouth, and the use of antisialagogues to decrease salivary flow. Most studies have shown that most sialoceles and fistulas will resolve within 2 to 3 weeks with this approach.

Secondary repair of the duct is generally difficult because of granulation and scar tissue. This runs the risk for delayed or poor wound healing as well as facial nerve injury. Radiation therapy has been used in the past but has been found to have a high failure rate in ductal injuries. More than 6 weeks is needed for gland atrophy to occur after radiation. There is also the risk of inducing malignancy, which has led to the abandonment of radiation for the treatment of sialoceles and cutaneous fistulas. Antisialagogues primarily have a role as an adjunct therapy to decrease salivary flow. Alone, they have had mixed reviews in the literature and have been found generally ineffective in major salivary duct injuries. Tympanic neurectomy has a high failure rate and does not speed recovery. Internalization of salivary flow could be considered for failures of conservative management. It has lower surgical risk than wound exploration and duct repair.

How well did you know this?
1
Not at all
2
3
4
5
Perfectly
31
Q

A 40-year-old man is evaluated for orbital fractures from a direct blow to the right eye. A swinging flashlight test is performed to evaluate a suspected afferent pupillary defect. Which of the following is the most appropriate description of this procedure?

A) The flashlight shone into the left eye causes both eyes to constrict; then when the light is shone in the right eye, the right eye constricts but the left eye does not
B) The flashlight shone into the left eye causes consensual constriction, and then both eyes dilate when the light is shone in the right eye
C) The flashlight shone into the left eye causes consensual constriction; then when the light is shone in the right eye, the right eye dilates while the left eye constricts
D) The flashlight shone into the left eye causes consensual dilation followed by paradoxical constriction when swung to the right eye
E) The flashlight shone into the left eye causes the left eye to constrict while the right eye remains dilated; both eyes are constricted when the light is shone in the right eye

A

The correct response is Option B.

A relative afferent pupillary defect, or Marcus Gunn pupil, results from an optic nerve dysfunction. The physical examination maneuver used to evaluate the optic nerve for a relative afferent pupillary defect is the swinging flashlight test described above. The most common cause of Marcus Gunn pupil is a lesion on the optic nerve, optic neuritis, or severe retinal disease but can also be caused by trauma. The normal response to light in one eye is constriction of both pupils and when the light is taken away both eyes dilate (consensual light reflex). In an abnormal exam the normal eye/optic nerve will cause a normal constriction of both pupils but the light shone in the affected eye will not have normal light perception (minimal pupillary constriction). The afferent pathway refers to the nerve impulse sent from the pupil to the brain via the optic nerve of one eye. The efferent pathway is the impulse sent back to both pupils via CN III and ciliary ganglia, causing both pupils to constrict.

How well did you know this?
1
Not at all
2
3
4
5
Perfectly
32
Q

A 21-year-old man is evaluated because of pain and double vision 2 weeks after being punched in the face. An orbital fracture is suspected. Which of the following characteristics of an orbital floor fracture is the most common indication for surgical repair?

A) Concomitant naso-orbito-ethmoid fracture
B) Contralateral zygomaticomaxillary complex fracture
C) Herniation of orbital contents with diplopia
D) Medial wall fracture without additional symptoms
E) Nondisplacement

A

The correct response is Option C.

The facial skeleton acts as a buffer and barrier against injury to the brain and deep structures. Reconstruction of the buttress system is integral to reforming the form and function of the facial skeleton.

Zygomaticomaxillary complex (ZMC) fractures, sometimes referred to as tripod fractures, may involve fractures of the following: zygomaticotemporal suture, maxillary sinus, and occasionally sphenoid bone. Correction involves reduction of the fracture, commonly with miniplate fixation or with 2-point wire fixation. These fractures can also involve the orbital floor and maxillary wall.

Symptoms may include numbness in the maxillary nerve (V2 of trigeminal (V) nerve), subconjunctival hemorrhage, diplopia, and enophthalmos. Orbital floor fractures can be isolated, as in a blowout, or involve other structures (e.g., orbital rim, orbital medial/lateral walls, orbital apex). Additional fractures are typically associated with higher-force injury and may require more extensive surgical repair.

Diagnostic study typically includes CT scan of the facial bones to assess the fractures and to delineate the extent of the facial trauma.

There are three main indications to treat isolated orbital fractures:

Entrapment of the extraocular muscles: this may cause injury and permanent dysfunction of the globe if not reduced.

Prevention of globe malposition: malposition of the globe may occur leading to diplopia or enophthalmos. This may not be apparent immediately after the injury because of soft-tissue edema. Reduction is indicated if there are signs of early enophthalmos, displacement of more than 50% of the orbital floor, or significant soft-tissue displacement.

Lateral orbital wall displacement: when the injury is more severe, the lateral orbital bone (sphenoid bone) may impact the orbital apex or even the mid-cranial fossa.

The CT scan should be examined for muscle entrapment, herniation of orbital contents, and associated fractures (naso-orbito-ethmoid, zygomaticomaxillary, Le Fort). Open reduction and internal fixation can be performed emergently or after edema has subsided, and usually after the orbit has been examined by an ophthalmologist for ocular injury.

Repair of the orbital floor can be done using several different types of materials—alloplastic and autologous. This is usually dictated by the surgeon’s preference and the extent of the injury.

How well did you know this?
1
Not at all
2
3
4
5
Perfectly
33
Q

A 50-year-old man is evaluated for facial trauma sustained after falling off a bicycle 5 days ago. The patient reports mid-facial pain and resolved swelling. Physical examination of the face shows no signs of trauma. Intraoral examination shows traumatic loss of tooth No. 8. He has no cervical spine tenderness or pain to active range motion. Which of the following is the most appropriate next step in management?

A) Obtain cervical spine MRI
B) Obtain maxillofacial CT scan to evaluate for facial fracture
C) Recommend elevation of the head of the bed and ice pack to the face
D) Recommend soft diet and referral to a dentist
E) No further management is necessary

A

The correct response is Option B.

The patient described most likely has a facial fracture and, thus, the most appropriate next step in management is to obtain a maxillofacial CT scan. A retrospective analysis of patients with facial trauma who had maxillofacial CT scan for the evaluation of facial fractures identified five independent criteria highly predictive of facial fracture. Any of the following five physical examination criteria – the Wisconsin criteria – were predictive of facial fracture: bony step-off or instability, periorbital swelling or contusion, Glasgow Coma Scale score less than 14, malocclusion, or tooth absence (sensitivity, 98.2%). Because the patient has a missing tooth, he should be evaluated with a maxillofacial CT scan. Although soft diet and eventual referral to a dentist as well as elevation of the head of bed with cool compress are reasonable recommendations, the first important step is obtaining a diagnosis.

Because the patient does not have tenderness of the cervical spine, there is no indication for obtaining cervical spine x-ray studies.

Systematic and efficient decision instruments in the management of multisystem trauma patients are critical in identifying all injuries including those of the maxillofacial skeleton and the cervical spine. The accurate use of systematic decision instruments could potentially result in a dramatic decrease in the number of CT scans and x-ray studies performed annually in the United States.

How well did you know this?
1
Not at all
2
3
4
5
Perfectly
34
Q

An 18-year-old woman is evaluated for persistent fullness of the left cheek 3 months after undergoing open reduction and internal fixation of a left zygomaticomaxillary complex fracture. She has no pain. The fracture was fixated at the inferior orbital rim (1.3-mm-thick titanium plate), the frontozygomatic suture (1.3-mm-thick titanium plate), and the lateral buttress (2.4-mm-thick titanium plate). On physical examination, the incisions are well healed. Which of the following is the most likely cause of this patient’s cheek fullness?

A) Excessive profile of the lateral buttress and orbital rim plate fixation
B) Malreduction of the fracture
C) Mucocele from the maxillary sinus
D) Residual soft-tissue edema
E) Subclinical hardware infection

A

The correct response is Option D.

Fullness of the soft-tissue envelope is expected after any severe trauma, regardless of location. This is further compounded by operative dissection to effect open reduction and internal fixation. Although most soft-tissue swelling around a fracture site dissipates in several weeks, there will be persistent thickening of the soft tissue in the area for many months after the trauma.

It is nearly impossible to “over-reduce” a zygomaticomaxillary (ZMC) fracture in the sagittal plane. Malreduction of these injuries characteristically results in flattening of the cheek (not fullness), vertical dystopia, or enophthalmos. Prominent hardware can create contour irregularities, but this would be localized and would not lead to global cheek fullness. Furthermore, the plate choices in areas where they could potentially be visible (i.e., the orbital rims) here are quite low profile and would be very unlikely to create excessive soft-tissue fullness. Hardware infection can rarely occur and lead to fullness of the cheek. Nevertheless, the absence of pain and the presence of healed wounds make this improbable. Mucocele from the maxillary sinus after repair of ZMC fracture is a rare but described entity and can result in localized soft-tissue fullness. These often occur in the context of a concurrent sinus infection and present many months to years after the injury. Soft-tissue swelling is usually accompanied by pressure and/or pain, and the symptoms can wax and wane. External drainage is not uncommon.

How well did you know this?
1
Not at all
2
3
4
5
Perfectly
35
Q

A 23-year-old man is brought to the emergency department 30 minutes after sustaining a self-inflicted shotgun wound to the face. Physical examination shows loss of soft tissue of the mid face with exposed mandible and maxilla, sonorous respiration, and periods of apnea. Heart rate is 100 bpm and blood pressure is 65/30 mmHg. Which of the following is the most appropriate course of management?

A) Assess and establish airway, control bleeding, perform secondary survey, stabilize cervical spine
B) Assess and establish airway, stabilize cervical spine, control bleeding, perform secondary survey
C) Assess and establish airway, stabilize cervical spine, perform secondary survey, control bleeding
D) Stabilize cervical spine, control bleeding, assess and establish airway, perform secondary survey
E) Take the patient to the operating room for debridement of facial wound with reconstruction

A

The correct response is Option B.

The appropriate management sequence in this patient is to assess and establish the airway, stabilize the cervical spine, control bleeding, and perform a secondary survey. Only after the patient’s condition has been stabilized using standard Advanced Trauma Life Support (ATLS) protocol would he be taken to the operating room for debridement and eventual facial reconstruction. While the deformity is obvious, and will require near-immediate attention in the operating theater, it can easily distract the evaluator from executing the ATLS management protocol.

The ATLS protocol was developed by the American College of Surgeons (ACS) Committee on Trauma (COT) in 1980 and is the standard of care for all trauma patients. The steps of the primary survey are remembered by the mnemonic ABCDE (Airway, Breathing, Circulation, Disability, Environment/exposure). The airway is the first priority and is assessed by determining the ability of air to pass unobstructed into the lungs. Treatment may require endotracheal intubation or establishment of a surgical airway. Breathing should then be evaluated to determine the patient’s ability to ventilate and oxygenate. Circulation is evaluated by identifying hypovolemia and external sources of hemorrhage. Disability is determined by performing gross mental status and motor examinations. The final step includes patient exposure and control of the immediate environment. The secondary survey should only be completed after following the fundamental steps of the ATLS protocol.

Cervical spine injury should be assumed in all trauma patients and should be managed as such until it can be definitively excluded. This patient with a gunshot wound to the face should have the cervical spine stabilized immediately and strict cervical spine precautions should be maintained during the assessment of the patient’s airway and breathing. During assessment of the airway, the cervical spine should not be flexed, extended, or rotated. If the external neck support must be removed, a member of the trauma team should maintain control of the head and neck using the in-line immobilization technique.

How well did you know this?
1
Not at all
2
3
4
5
Perfectly
36
Q

A 21-year-old man is evaluated after being struck in the chin with a lead pipe during a bar fight. Physical examination shows a 6-mm displaced bicortical right parasymphyseal fracture and left nondisplaced subcondylar fracture with open crossbite. A full trauma evaluation is performed; no injury to the cervical spine or intracranial injury is noted. Which of the following is the most appropriate treatment in this patient?

A) Liquid diet for 3 weeks, followed by mechanical soft diet for 3 weeks
B) Maxillomandibular fixation followed by open reduction and internal fixation (ORIF) in 4 weeks
C) Maxillomandibular fixation alone for 4 to 6 weeks
D) ORIF of the parasymphyseal fracture followed by immediate maxillomandibular fixation
E) Maxillomandibular fixation followed by immediate ORIF of the parasymphyseal fracture

A

The correct response is Option E.

The proper surgical repair is rigid maxillomandibular fixation, followed by open reduction and internal fixation (ORIF) of the parasymphyseal fracture. Patients with mandibular fractures sustain significant blunt force trauma and require a full trauma assessment. Closed head injury and cervical spine trauma must be excluded. Additionally, a thorough assessment of the facial skeleton for other fractures is necessary.

Mandibular fracture repair is based on the paramount principle of restoring proper dental occlusion. As such it is necessary to place the patient into maxillomandibular fixation (MMF), rigidly with arch bar technique prior to addressing the parasymphyseal fracture. MMF will effectively reduce the subcondylar fracture and realign the bicortical anterior fracture, allowing plate and screw fixation to be done anatomically. Rigid fixation is needed in this particular case due to the need for full immobilization of several weeks while the subcondylar fracture sets and heals.

Mandibular fracture rigid fixation with plates and screws without proper restoration of dental occlusion can potentially lead to severe consequences. Patients will have temporomandibular joint pain due to the malalignment of the maxilla and mandible. They can develop associated painful trismus. Nonunion with or without osteomyelitis can also occur due to the force of the muscles of mastication and the abnormal mandibular forces created by the malocclusion.

Starting the surgical repair with ORIF of the parasymphyseal fracture followed by MMF is inappropriate since proper dental occlusion is not assured prior to the rigid fixation. With the presence of the subcondylar fracture, the MMF placed after plating the parasymphyseal fracture can still achieve dental occlusion by further displacement of the subcondylar fracture, rather than reducing it.

MMF with elastics followed by open repair of the parasymphyseal fracture in 4 weeks is inappropriate. Because of displacement of both fractures, rigid MMF is needed. Furthermore, waiting 4 weeks to reduce and fixate the parasymphyseal fracture will likely lead to fibrous non-union or malunion. Facial fractures in adults should be repaired within 10 to 14 days so as to readily manipulate the fracture elements for anatomical reduction.

MMF with elastics as the only treatment is also inappropriate for the treatment as indicated above. Similarly liquid and soft mechanical diets without fixation is inappropriate and will lead to nonunion or malunion with subsequent malocclusion and pain.

How well did you know this?
1
Not at all
2
3
4
5
Perfectly
37
Q

A 52-year-old man is evaluated 7 months after sustaining an isolated left-sided orbital blowout fracture for which no surgical intervention was performed. Physical examination shows extraocular movements are intact. He reports gradual and progressive episodes of double vision over the past 3 months. Which of the following physical findings is most likely associated with this condition?

A) Ectropion
B) Muscle entrapment
C) Exophthalmos
D) Vertical dystopia
E) Orbital proptosis

A

The correct response is Option D.

The physical finding that would most likely be associated with the finding of double vision 7 months after having sustained an orbital blowout fracture is enophthalmos. The position of the globe is altered in enophthalmos. Because of increased orbital volume, the globe is situated deeper or more inferiorly, such that the direct line of vision of the affected eye no longer matches that of the unaffected eye. The two eyes no longer track appropriately, causing double vision or diplopia by virtue of not having the same visual field.

Orbital fractures frequently occur in conjunction with other facial fractures, most commonly the zygomaticomaxillary complex fracture. Often the orbital floor or sidewall can be fractured without concomitant fractures. This occurs from a direct blow to the orbit/eye, most often by a punch or elbow/ball in sports. The absolute indication for operative repair acutely is the presence of entrapment, most often of the inferior rectus muscle. The belly of the muscle herniates into and remains within the fracture site in the orbital floor. The finding of diplopia on upward, lateral gaze with the affected eye having reduced movement in that upward direction indicates entrapment. The forced duction test further documents this condition. Surgical repair is indicated acutely to avoid permanent damage and fibrosis to the inferior rectus muscle and consequent visual difficulties.

In the absence of entrapment or orbital hematoma, surgical repair is a judgment call based on the location and extent of the fractures. If the fracture is deemed sizable enough to increase the orbital volume, then the risk of subsequent enophthalmos and diplopia is considered high and surgical repair is indicated. The repair involves reduction of orbital contents and restoration of the orbital floor, most often with titanium mesh. Large fractures may require primary bone grafting.

The conventional means of correcting post-traumatic enophthalmos is bone grafting. However, it is often difficult to accurately judge the correction needed due to multimodal increase in orbital volume from the combination of increase orbital volume and potential reduction in periorbital fat from traumatic atrophy. New techniques of alloplastic floor restoration and periorbital fat grafting have shown encouraging results.

Exophthalmos, ectropion, and proptosis are not conditions typically associated with isolated orbital blowout fractures. Furthermore, these conditions frequently occur without diplopia.

Entrapment of the periorbital muscles, most usually the inferior oblique, can cause entrapment by limiting proper movement of the involved globe. However, this is usually an acute symptom and would not present initially several months after the trauma.

How well did you know this?
1
Not at all
2
3
4
5
Perfectly
38
Q

In patients without loss of posterior vertical height, which of the following is the most appropriate duration of maxillomandibular fixation (MMF) for treatment of minimally displaced bilateral subcondylar fractures of the mandible?

A) 1 week
B) 4 weeks
C) 8 weeks
D) None; stabilize mandible with gunning splint for 8 weeks
E) None; treat patient with soft diet

A

The correct response is Option B.

Closed reduction has historically been the standard treatment for subcondylar fractures of the mandible. Its widespread use is attributed to the idea that closed reduction results in fewer complications with similar functional and aesthetic outcomes compared with open reduction and internal fixation (ORIF). For instance, complications such as facial nerve damage and excessive scarring are significantly decreased because of the noninvasive nature of this approach. However, as highlighted by ongoing debate, a consensus regarding outcomes between open and closed reduction is not evident in the literature. In short, some studies conclude that both approaches produce roughly similar results, while other studies have associated an array of unfavorable outcomes with closed reduction. These outcomes include facial asymmetry, deviation upon mouth opening, skeletal malocclusion, and chronic pain of the temporomandibular joint (TMJ). The fact that many of these parameters lack standardization in the duration of the treatment further obscures the debate. Larger studies with consistent parameters are needed to reassess outcomes with the surgical techniques and technology present today. However, it is unlikely that a large enough trial will deliver granular evidence to conclusively quell this debate.

Another controversial point regarding closed reduction is the length of time a patient should spend in maxillomandibular fixation (MMF). Many surgeons choose to apply fixation for a very short period (i.e., 2 weeks) to avoid ankylosis of the TMJ secondary to forced immobilization during MMF. While the etiology of ankylosis is not completely understood, it is hypothesized that trauma leading to intracapsular hematoma results in fibrosis and excessive bone formation, ultimately causing hypomobility of the affected side.

Given the current hypothesis, ankylosis of the TMJ is most likely a manifestation of direct injury within the joint capsule or condylar head itself. It is imperative to point out that as a result, there should be a lower risk of ankylosis in subcondylar fractures compared with fractures of the condylar head. Therefore, the position of the fracture line relative to the joint capsule should be closely examined, and a longer period of MMF should be employed if there is no involvement of the condylar head, disc, or capsule. A longer period of MMF commonly results in better union of the fractured segments with no increase in the incidence of ankylosis. In a nondisplaced fracture or minimally displaced fracture with a functional occlusion, 4 to 6 weeks of MMF is recommended, followed by 2 to 3 weeks of guiding elastics. The same treatment applies in the case of a nondisplaced bilateral fracture. However, this scenario is less common because the force parameters to cause the bilateral fracture are often greater and tend to displace the fracture fragments significantly, necessitating ORIF. The patient will most likely develop shortening of the masseter muscle and stiffness with prolonged immobilization during 8 to 10 weeks. With bilateral instability, the fracture pattern is unstable and soft diet alone would likely lead to malocclusion. A gunning splint is often used in edentulous mandible only and is not functional MMF.

How well did you know this?
1
Not at all
2
3
4
5
Perfectly
39
Q

Which of the following concomitant fractures is most likely to affect the perceived reduction when performing open reduction and internal fixation of a zygomaticomaxillary complex fracture?

A) Anterior maxillary wall comminution with loss of bone
B) Articular tubercle of the zygomatic arch
C) Le Fort I fracture
D) Naso-orbito-ethmoid fracture
E) Orbital floor blowout fracture

A

The correct response is Option D.

When surgically reducing a zygomaticomaxillary complex (ZMC) fracture (tripod fracture), the surgeon uses multiple landmarks to confirm adequate reduction. The lateral orbital sidewall is the most reliable landmark as it is a three-dimensional landmark (junction of frontal bone, sphenoid, and zygoma). Other landmarks include the zygomaticofrontal suture, the zygomaticomaxillary lateral buttress, the orbital rim, and the zygomatic arch. Because the orbital rim is easily visualized, surgeons may rely heavily on this landmark for alignment. Sometimes due to orbital swelling or poor visualization, the lateral orbital wall is not easy to assess for adequate reduction of the fracture. If the orbital rim appears to be reduced (well aligned) but the ZMC fracture is not well reduced, then an ipsilateral unreduced naso-orbito-ethmoid (NOE) fracture is the most likely reason. It is important to reduce the NOE fracture before aligning the orbital rim for the ZMC fracture reduction. Postoperative deformity will ensue if the NOE fracture is not reduced and the ZMC is plated in position based on the orbital rim alignment. The deformity will lead to enlarged orbital volume and facial widening, loss of malar projection, and enophthalmos. Proper reduction and initial management are important, as secondary corrections are more difficult and have more morbidity.

How well did you know this?
1
Not at all
2
3
4
5
Perfectly
40
Q

Isolated orbital fractures most commonly occur in which of the following bones?

A) Ethmoid
B) Frontal
C) Lacrimal
D) Maxillary
E) Zygomatic

A

The correct response is Option D.

Most isolated orbital fractures involve the orbital floor, a majority of which is made up of the maxillary bone. The maxillary bone is quite thin behind the infraorbital rim, and is perforated by the infraorbital nerve passing in a canal below it. Most pure blowout fractures involve the orbital floor.

Long-term epidemiologic data regarding the natural history of orbital bone fractures are important for the evaluation of existing preventive measures and for the development of new methods of injury prevention and treatment.

How well did you know this?
1
Not at all
2
3
4
5
Perfectly
41
Q

A 24-year-old man is brought to the emergency department after being ejected from a vehicle at high speed. Physical examination shows massive oronasal bleeding and an unstable maxilla. He is hemodynamically unstable, and other sources of marked bleeding have been excluded. Endotracheal intubation is performed. Which of the following is the most appropriate next step in management?

A) Establishment of mandibulo-maxillary fixation
B) Nasendoscopy with bipolar coagulation
C) Operative ligation of the external carotid arteries
D) Placement of anterior and posterior nasal packing
E) Transcatheter embolization

A

The correct response is Option D.

After establishing an airway, the best first step to controlling massive oronasal hemorrhage is nasal packing. This can be quickly accomplished by inflating Foley catheters in the posterior choanae, followed by anterior packing with either a nasal tampon or ribbon gauze. Nasal packing has been shown to control bleeding in 29% of such patients and decrease it in another 44% of patients.

Establishment of mandibulo-maxillary fixation (MMF) may also control oronasal hemorrhage. However, achieving MMF in the emergency department is often limited by the availability of fixation devices and is complicated by the presence of an endotracheal tube.

Emergent transcatheter arterial embolization, when available, is highly effective in identifying and controlling oronasal hemorrhage when packing has failed to do so. The internal maxillary and superficial temporal arteries are most often responsible for such bleeding.

Operative ligation of the external carotid arteries is rarely effective to control oronasal hemorrhage due to rich collateral blood flow in the head and neck.

The utility of nasendoscopy is limited in the presence of marked bleeding as visualization is poor and the bleeding vessel may not be readily visible.

How well did you know this?
1
Not at all
2
3
4
5
Perfectly
42
Q

A 38-year-old truck driver is examined in the emergency department following a motor vehicle collision. He is sedated and intubated on a spine board. On examination, ecchymoses over the cheeks, swelling over the left jaw, and pulsation of the left globe are noted. Which of the following is the potential fracture of most concern?

A) Mandibular body
B) Medial maxillary buttress
C) Nasal bone
D) Orbital roof
E) Zygomatic-malar complex

A

The correct response is Option D.

Fractures of the orbital roof that enter the middle cranial fossa may allow communication between the cavernous sinus and the carotid artery. Other findings include associated bruit and ipsilateral blindness, which would not be appreciated in the obtunded patient. The remaining fractures are important but are not as critical as a carotid-cavernous fistula.

How well did you know this?
1
Not at all
2
3
4
5
Perfectly
43
Q

Proper reduction of an isolated zygoma fracture requires reduction and realignment of which of the following?

A) Zygomaticofrontal suture, zygomaticomaxillary buttress, and infraorbital rim
B) Zygomaticofrontal suture, zygomaticomaxillary buttress, and orbital floor
C) Zygomaticofrontal suture, zygomaticonasal suture, and infraorbital rim
D) Zygomaticomaxillary buttress, infraorbital rim, and nasomaxillary buttress
E) Zygomaticomaxillary buttress, orbital floor, and alveolus

A

The correct response is Option A.

A zygoma fracture involves displacement of the zygoma that articulates with the frontal bone, maxilla, and sphenoid. In order to stabilize the fracture after adequate reduction, the zygomaticofrontal, zygomaticomaxillary buttress, and infraorbital rim need to be stabilized. If there is a large (>2 cm2) defect in the orbital floor after reduction, reconstruction of the orbital floor is also necessary to prevent enophthalmos.

Although the nasomaxillary buttress is one of the vertical buttresses of the face, the zygoma does not articulate with the nasal bones.

How well did you know this?
1
Not at all
2
3
4
5
Perfectly
44
Q

A 23-year-old man sustains multiple fractures to the middle and upper face during a motor vehicle collision. The patient reports disturbances of smell immediately afterwards. Injury to which of the following bones is the most likely reason for this patient’s anosmia?

A) Ethmoid
B) Frontal
C) Nasal
D) Sphenoid
E) Vomer

A

The correct response is Option A.

The cribriform plate is the horizontal component of the ethmoid bone which supports the olfactory bulb and creates a passageway for the olfactory nerves.

The cribriform plates (right and left) have many foramina that act as passageways for olfactory nerves and are in intimate contact with the meninges. In addition to anosmia (loss of smell), injury to the cribriform plate may cause tearing of the meninges with cerebrospinal fluid leakage. The crista galli is a midline prominence of the ethmoid, immediately above the cribriform plates, which serves as a point of attachment for intracranial soft tissue. Other components of the ethmoid bone are the vertical portion and the lateral masses. The vertical portion is called the perpendicular plate, which forms part of the nasal septum. The lateral masses of the ethmoid bone contain a plate of bone called the lamina papyracea, which forms part of the medial orbital wall and the ethmoid air cells.

While anosmia and taste abnormalities can occur following a variety of facial fracture patterns, including frontal bone, naso-orbital-ethmoid, nasal, Le Fort, and zygoma, the close anatomic relationship of the ethmoid cribriform plate to the olfactory nerve makes it most likely.

How well did you know this?
1
Not at all
2
3
4
5
Perfectly
45
Q

A 21-year-old woman comes to the emergency department because she has had ocular pain and decreased vision since she was struck in the face with a bottle 2 hours ago. On examination in a dark room, both pupils constrict when a light is shone in the right or left sides directly. When the light is shifted from the right to the left, the pupils dilate. Which of the following is the most likely pathologic condition in this patient?

A) Central retinal artery occlusion
B) Oculomotor (III) nerve injury
C) Preexisting cataract
D) Symmetrical acute glaucoma
E) Vitreous hemorrhage

A

The correct response is Option A.

A Marcus Gunn pupil is a relative afferent pupillary defect caused by a lesion of the optic nerve (between the retina and the optic chiasm) or severe retinal disease. It is observed during the swinging-flashlight test whereupon the patient’s pupils constrict less (therefore appearing to dilate) when a bright light is swung in front of the unaffected eye to the affected eye. The affected eye still senses the light and produces pupillary sphincter constriction to some degree, albeit reduced. Conditions that do not cause a Marcus Gunn pupil include cataracts, vitreous hemorrhage, injury to the oculomotor nerve (cranial nerve III), or symmetrical acute glaucoma.

How well did you know this?
1
Not at all
2
3
4
5
Perfectly
46
Q

A 23-year-old man is brought to the emergency department unconscious after a head-on motor vehicle collision. Physical examination shows left periorbital bruising. When a light is shined into the left eye, no pupillary constriction is noted, and thereafter, the light is shined into the right eye, and both pupils constrict. When the light is subsequently shined again into the left eye, the right pupil constricts and the left pupil dilates. Which of the following best explains this finding?

A) Extraocular muscle entrapment
B) Hyphema
C) Inadequate brain perfusion
D) Optic nerve injury
E) Tentorial herniation

A

The correct response is Option D.

The vignette illustrates findings consistent with a Marcus Gunn pupil. A lesion at the level of the posterior globe (retina) and optic nerve anterior to the chiasm alters the afferent response normally expected from shining a light in the eye. The test is done by shining a light in the pupil of the affected side first, and this will cause little or no constriction on that side, yet there will be a consensual response on the normal side. Then, the light is shined in the normal pupil, resulting in constriction of both pupils (normal ipsilateral and consensual response). The third step is to swing the light back to the injured side, and this typically will cause paradoxical dilatation of the affected pupil. A fracture in the posterior orbit may cause pressure on the optic nerve directly by impingement of a fracture fragment or from hematoma.

Extraocular muscles control the directionality of the globe for the purpose of binocular vision. There is no direct distal anatomical connection to the pupillary sphincter muscle (parasympathetic) or the iris dilator (sympathetic), both of which control pupillary size. However, there is a proximal commonality in that some extraocular muscles (superior/medial/inferior recti, inferior oblique) and the pupillary muscles are supplied by the oculomotor nerve. Most commonly, it is the inferior oblique with or without the inferior rectus muscle that is entrapped in an orbital floor fracture, and thus has no direct bearing on pupillary size. Therefore, pupils should react normally in an isolated orbital floor fracture with muscle entrapment.

Hyphema is the presence of blood in the anterior chamber. It does not cause a Marcus Gunn pupil, or any other abnormal pupillary reaction.

The pupils in a patient with inadequate brain perfusion are usually dilated and fixed, or react sluggishly to light, but always symmetrical.

In tentorial herniation, due to a laterally expanding intracranial hematoma, the oculomotor nerve is susceptible to compression as it exits the mid brain. The pupillary examination will show one ipsilateral dilated pupil that is fixed or sluggish, and the globe will be laterally displaced due to the unopposed tone generated by cranial nerve VI on the lateral rectus muscle over a paralyzed medial rectus muscle.

Normal pupillary constriction as a reaction to light begins with a light stimulus on the retina that triggers a signal via afferent fibers in the optic (II) nerve. These fibers connect to the pretectal nucleus and both Edinger-Westphal nuclei. A response is generated from there through parasympathetic fibers via the oculomotor (III) nerve, which supplies the sphincter pupillae.

How well did you know this?
1
Not at all
2
3
4
5
Perfectly
47
Q

A 35-year-old man is evaluated because of pain in one of the right lower molar teeth 6 months after he underwent open reduction and internal fixation of a right mandibular angle fracture. Examination shows grayish discoloration and tenderness to axial percussion in the mandibular second molar. A panoramic x-ray study (Panorex) shows an intact mandibular second molar with 1.5-cm radiolucency at its apex. Which of the following is the most likely cause of these findings?

A) Dentigerous cystnsufficiency
B) Mucous retention cyst
C) Odontogenic keratocyst
D) Periapical cyst
E) Traumatic bone cyst

A

The periapical cyst is the most common odontogenic cyst. The usual etiology is a nonviable tooth that becomes infected, leading to necrosis of the pulp. Toxins exit the apex of the tooth, leading to periapical inflammation leading to a radiolucency seen on x-ray study.

The dentigerous cyst is the second most common odontogenic cyst which develops within the normal dental follicle surrounding an unerupted tooth such as a mandibular or maxillary third molar or maxillary canine. Most are asymptomatic and found incidentally on x-ray study.

The odontogenic keratocyst or OKC is the third most common odontogenic cyst. It is a great mimic and can have a wide variety of clinical presentations. These cysts are rapid growing and aggressive and can be difficult to remove; recurrence rates are high. They are also a component of the basal cell nevus syndrome.

Mucous retention cyst, or mucocele, is a pseudocyst that arises from trauma to the minor salivary glands in the lips.

Traumatic bone cyst is a self-limiting radiolucent lesion of uncertain etiology that forms an empty or fluid-filled cavity most commonly within the mandible but also elsewhere in the body.

How well did you know this?
1
Not at all
2
3
4
5
Perfectly
48
Q

A 25-year-old man comes to the office for evaluation of unilateral proptosis, visual impairment, and limitation of ocular movements. History includes basilar skull fractures and repair of panfacial fractures 4 weeks ago. On physical examination, the left eye appears to be pulsating. Which of the following is the most appropriate next step?

A) Beta-2 transferrin assay
B) Carotid duplex
C) Cerebral angiography
D) Craniotomy
E) Noncontrast CT scan

A

The correct response is Option C.

This case described represents the classic presentation of a posttraumatic carotid-cavernous fistula (CCF). Symptoms include pulsatile proptosis, ocular and orbital erythema, chemosis, diplopia, headaches, and visual loss. This pathology occurs through abnormal connections between the internal carotid artery and the cavernous sinus, and, on rare occasions, may appear between the internal carotid or branches of the external carotid artery and the venous plexus of the skull. Traumatic CCF can lead to blindness as the disorder progresses and, in rare cases, can result in paralysis, unconsciousness, and even death. It is important that clinicians operating on the craniofacial region, and especially those who manage craniofacial trauma, have a thorough understanding of this potentially lethal entity. Although CCF occurs rarely after craniofacial trauma, this disorder is thought to occur relatively frequently in patients with basilar skull fractures. Once there is a suspicion, a prompt evaluation of the arterial vasculature around the cavernous sinus is required. Most commonly, a cerebral angiogram is used to make the diagnosis and, using related techniques, a variety of curative measures (e.g., embolization) can then be enacted. A craniotomy-type modality would be relegated to only the most refractory of cases and would be for cure and not diagnosis. A noncontrasted CT scan would only show posttraumatic bony derangements. Beta-2 transferrin assay could be used if there is an accompaniment of clear rhinorrhea to rule out a craniospinal fluid leak. CCF can be mistaken for other pathologies such as an orbital apex syndrome or even stroke. The latter suspicion may lead to the ordering of a carotid duplex, which would be expected to be normal.

How well did you know this?
1
Not at all
2
3
4
5
Perfectly
49
Q

An 18-month-old girl is brought to the office because of a soft spot with pulsations on the right parietal region of her skull. At age 6 months, the patient sustained a linear skull fracture due to head trauma, which was managed nonoperatively. CT scan shows a growing skull fracture. Which of the following is the most likely cause of this patient’s growing fracture?

A) Dural injury at the time of the fracture
B) Genetic bone healing disorder
C) Isolated right unicoronal craniosynostosis
D) Patent anterior fontanelle
E) Trauma-related infection

A

The correct response is Option A.

Growing skull fractures occur in children. If a child sustains a fracture that appears linear but has an underlying dural injury, then the fracture may fail to heal. As the skull continues to grow, so does the cranial defect.

Trauma alone, open fontanelles, genetic bone disorders, and craniosynostosis are not associated with growing skull fractures.

Children under 2 years of age have a tremendous ability to regenerate bone and heal fractures or even large defects. Having normal, healthy dura is very important to bone healing.

How well did you know this?
1
Not at all
2
3
4
5
Perfectly
50
Q

A 14-year-old boy is brought to the emergency department after a fall. Physical examination shows ocular entrapment. A CT scan is shown. Surgical correction is planned. Which of the following is the most appropriate location for incision in this patient?

A) Bicoronal
B) Lateral brow
C) Orbital rim
D) Subciliary
E) Transcaruncular

A

The correct response is Option E.

The CT scan shows an isolated medial wall fracture with entrapment of the medial rectus muscle. The transcaruncular approach can be used for isolated medial wall surgery or combined with a retroseptal transconjunctival approach to the orbital floor (with or without a lateral canthotomy).

Although a bicoronal incision will give the necessary exposure to reduce the entrapment and repair the fracture, it has a higher morbidity associated with it, primarily in regards to blood loss and scarring. In this case, it would be excessive in regards to what is needed to expose and treat an isolated medial wall fracture. The bicoronal incision is more appropriate in complex cases where multiple fractures are involved or if access to a naso-orbital-ethmoid or frontal sinus fracture is needed.

In this case, subciliary, orbital rim, or lateral brow incisions would not give adequate exposure to reduce the entrapped muscle and repair the fracture.

How well did you know this?
1
Not at all
2
3
4
5
Perfectly
51
Q

A 27-year-old man is brought to the emergency department after he sustained a small-caliber gunshot wound through the cheek, obliterating the intraoral orifice of the parotid duct. Which of the following is the most appropriate management?

A) Ligation
B) Primary repair
C) Secondary intention healing
D) Stenting
E) Superficial parotidectomy

A

The correct response is Option D.

It is recommended that injury to the orifice of the parotid duct be managed with a stent or a drain to maintain patency during healing and guide the flow of saliva into the mouth.

Allowing the ductal injury to heal secondarily will likely result in blockage of the orifice with scar tissue, which could lead to a sialocele or cutaneous fistula.

Ligation of the proximal duct is recommended for extensive injuries of the glandular/duct system in which neither end of the duct is amenable to repair. This will result in eventual atrophy and loss of function of the parotid gland.

Primary repair alone would not be recommended given the degree of soft-tissue loss and contusion with this mechanism of injury.

Superficial parotidectomy would be considered for management of chronic glandular/duct system fistulas.

How well did you know this?
1
Not at all
2
3
4
5
Perfectly
52
Q

A 33-year-old man is brought to the emergency department after he is struck in the forehead with a baseball bat. The patient is awake and alert. Examination shows an obvious deformity in the frontal region. Emergency CT scan shows a displaced and comminuted fracture of the anterior wall of the frontal sinus and a nondisplaced fracture of the posterior wall. The frontonasal duct is patent. Which of the following is the most appropriate treatment?

A) Cranialization of the sinus
B) Craniotomy
C) Obliteration of the sinus and fat grafting
D) Observation and intravenous administration of antibiotics
E) Open reduction and internal fixation of the anterior wall of the frontal sinus

A

The correct response is Option E.

Frontal sinus fractures can be assessed by patency of the nasofrontal duct and by whether the fracture involves the anterior wall of the sinus, the posterior wall, or both. In this case, CT scanning shows significant damage to the anterior wall and a minimal fracture in the posterior wall. The normal nasofrontal duct would allow salvage of the sinus with minimal risk of a mucocele or an infection. The underlying brain is uninvolved, so craniotomy is unnecessary. The obvious deformity indicates open reduction and fixation, and because the posterior wall is minimally involved, repair of the anterior wall without craniotomy could be performed.

How well did you know this?
1
Not at all
2
3
4
5
Perfectly
53
Q

A 5-year-old boy is evaluated because of downward and lateral displacement of the left globe and diplopia that has increased steadily for the past 18 months. History includes open reduction and internal fixation of a frontal bone fracture at 2 years of age. An image is shown. This deformity was most likely caused by which of the following at the time of repair?

A ) Failure to obliterate the frontal sinus
B ) Failure to recognize dural injury and insufficiency
C ) Failure to repair periorbital lacerations
D ) Poorly aligned bony fixation
E ) Use of absorbable plates

A

The correct response is Option B.

Head injuries in infants and young children accompanied with skull fracture often contain dural laceration. Although rare in occurrence, when these dural tears are not recognized (especially when extensive), a growing skull fracture (GSF) can occur. Repair of these dural tears acutely prevents the development of GSF and the potential for secondary cortical injury. The frontal sinus, unlike the maxillary and ethmoid sinuses, is not present at birth and only starts to develop at age 2 years, which, in this case, was when the original repair was performed. Thus, obliteration of the sinus was not warranted. Lacerations of the periorbita, poor bone fixation, and the use of absorbable plates are not linked to GSF.

How well did you know this?
1
Not at all
2
3
4
5
Perfectly
54
Q

A 25-year-old man comes to the office 12 hours after sustaining a maxillofacial injury during a bicycle collision. Physical examination shows that the injured tooth is stable but sensitive to cold liquids, hot liquids, and air. No other abnormalities are noted. The patient?s pain suggests a fracture extending into which of the following?

A ) Alveolar bone
B ) Cementum
C ) Dentin
D ) Enamel
E ) Pulp

A

The correct response is Option C.

Dental trauma can result from a variety of mechanisms. Significant trauma should be evaluated completely in the emergency department, but injury that is isolated to the teeth can be managed in an outpatient setting. Tooth fractures can be minor, involving only the outer enamel layer, or more significant. Beneath the enamel is the dentin. This yellowish substance overlays and protects the inner tooth pulp. The visible and exposed one third of the tooth is the crown; it consists of outer enamel, inner dentin, and a small amount of the pulp. The two thirds of the tooth in the alveolar bone is called the root and consists of outer cementum, inner dentin, and the pulp chamber.

X-ray studies are useful to diagnose the extent of a dental facture. The sensitivity to cold and air that the patient is experiencing suggests a deeper fracture extending into the dentin. Prolonged exposure of the dentin is not only painful but also risks infection that can progress into the pulp and destroy the tooth. The tooth should be capped soon, and follow-up x-ray studies should be performed in 3 to 4 months to make sure that the pulp has not died.

Treatment of an injury to the pulp itself would require extracting the dying tooth and replacing it with a prosthetic. Alternatively, a root canal — removing all the dying pulp and filling the tooth — can be performed to avoid extraction. Fractures of the alveolar bone cause the teeth to be very unstable. An arch wire is applied while the bone heals. The tooth is then assessed with x-ray study to make sure it is viable. A root canal may be required in the future.

How well did you know this?
1
Not at all
2
3
4
5
Perfectly
55
Q

A 23-year-old male pitcher is struck in the face with a baseball during practice and sustains an isolated blowout fracture of the medial orbital wall. A transcaruncular approach is chosen for exposure. After incising the medical conjunctiva, dissection is best carried out between which of the following two structures to reach the posterior lacrimal crest?

A ) Inferior rectus muscle and capsulopalpebral fascia
B ) Lockwood ligament and inferior oblique muscle
C ) Medial canthal ligament and lacrimal duct
D ) Medial orbital septum and Horner muscle
E ) Whitnall ligament and medial rectus muscle

A

The correct response is Option D.

The appropriate plane of dissection is between the medial orbital septum and Horner muscle to expose the periosteum immediately posterior to the posterior lacrimal crest. This approach attempts to minimize injury to the Lockwood ligament which is more inferior and the lacrimal sac which is more anterior. Preservation of the septum will minimize spillage of fat into the surgical field.

How well did you know this?
1
Not at all
2
3
4
5
Perfectly
56
Q

A 45-year-old woman is evaluated because of enophthalmos caused by trauma to the left orbit sustained 6 months ago. Which of the following is the minimum increase of orbital volume that can result in post-traumatic enophthalmos?

A ) 1%
B ) 5%
C ) 10%
D ) 25%
E ) 50%

A

The correct response is Option B.

If the anatomical volume of the bony orbit is increased as a result of fracture displacement following trauma or inadequate restoration of the normal anatomy, patients may develop enophthalmos. This may be seen with an increase in volume in excess of 5% and may be commonly encountered following fracture of the orbital floor or the medial wall of the orbit.

How well did you know this?
1
Not at all
2
3
4
5
Perfectly
57
Q

A 28-year-old man comes to the office for consultation regarding correction of a residual deformity of the right infraorbital region 2 years after undergoing open reduction and internal fixation and repair of lacerations for an open orbitozygomatic fracture. Examination shows infraorbital soft-tissue atrophy. No enophthalmos or bony stepoff is noted. Examination of the eyes shows no abnormalities. CT scan shows healed fractures with good bony alignment. Which of the following is the most appropriate surgical intervention for correction of this deformity?

A ) Alloplastic prosthesis
B ) Fat grafting
C ) Rhytidectomy
D ) Rib grafting
E ) Scapular flap coverage

A

The correct response is Option B.

The patient has a small area of soft-tissue atrophy, which in the context of his history is likely due to fat atrophy. His bony structure is adequate and symmetrical, but he lacks subcutaneous fat. For this reason, restoration of the defect is best approached by replacing missing soft tissues, rather than enhancing the support framework. The localized nature of the defect makes it ideal for an injectable soft-tissue filler. When harvested and transferred correctly, fat grafting has shown to have high percentage rates in permanency, serving well as a soft-tissue filler. Other fillers that are based with collagen, hyaluronic acid, calcium hydroxyapatite, and poly-L lactic acid have the drawback of being temporary, but could also be considered in the absence of donor areas for fat graft.

Alloplastic prostheses are more suited for framework deficiencies. Besides, the unique configuration of the contour deficiency in the scenario described makes it less likely that a pre-shaped prosthesis is going to provide symmetry. A prosthesis in this case would be very palpable, due to the thin soft-tissue covering. Alloplastic prostheses are more likely to cause infection and seroma compared with autologous tissue. A rhytidectomy would not be indicated in this case because it is a procedure that is designed to reposition and/or suspend displaced soft tissues. The contour defect would most likely persist following rhytidectomy.

Rib grafts have been used for the correction of late deformities following inadequate treatment of depressed orbitozygomatic fractures. The aim is to reestablish facial projection as determined by the degree of depression of the skeletal framework. Rib grafts are not indicated as a soft-tissue filler.

Free scapular flaps are too bulky for the limited size defect of this patient’s face. However, for a larger defect, such as in Romberg disease, it provides suitable correction.

58
Q

A 72-year-old man is brought to the emergency department after he sustained injuries in a high-speed motor vehicle collision as an unrestrained backseat passenger. He has chronic obstructive pulmonary disease and a 40-pack-year history of smoking. The following measurements are obtained:

Heart rate 88 bpm

Respirations 18/min

Blood pressure 115/70 mmHg

Oxygen saturation 98% on 6 L by face mask

Physical examination shows severe swelling in the face. He is coughing blood and mucus from his mouth and nose. Gross malocclusion is noted, but full dentition is present with no dental caries. CT scan shows a naso-orbital-ethmoid fracture, Le Fort III fracture, palatal fracture, and comminuted mandibular body and angle fractures. Which of the following is the most appropriate method of airway management during surgical repair of this patient’s fractures?

A) Cricothyroidotomy
B) Nasotracheal intubation
C) Placement of an orotracheal tube
D) Tracheostomy
E) Use of a laryngeal mask airway

A

The correct response is Option D.

The method of airway management in patients with facial fractures can be controversial and should be individualized. The patient described has complex facial fractures involving both the midface and the lower face. He also has a significant history of smoking. This particular patient is likely to have continued respiratory issues postoperatively, making pulmonary management challenging. The placement of a tracheostomy at the time of surgery will allow the surgical team full access to all of the patient’s facial fractures and will facilitate the patient’s pulmonary care postoperatively.

Cricothyroidotomy is indicated occasionally as an emergency procedure when there is concern for acute control of the patient’s airway. The patient described is hemodynamically stable and is not in respiratory distress.

Nasotracheal intubation is contraindicated in a patient with a naso-orbital-ethmoid fracture because the presence of a tube can complicate fracture reduction.

Generally, placement of an orotracheal tube is feasible and successful in most facial fracture patients. However, given the complex nature of fractures in the scenario described, the patient will need to be placed into mandibular-maxillary fixation during surgery to obtain normal occlusion and possibly for an indefinite period of time after surgery. Although the tube can occasionally be placed behind the last molar or through a gap where there is a missing tooth, an orotracheal tube can make it difficult to obtain normal occlusion. It is not as beneficial as tracheostomy in providing the postoperative pulmonary care that will likely be required in the patient described. A laryngeal mask airway does not provide as secure an airway as either orotracheal intubation or tracheostomy. During a complex facial fracture surgery, the head may need to be manipulated or turned, which could potentially dislodge a laryngeal mask airway.

59
Q

A 22-year-old man is brought to the emergency department after sustaining injuries during an all-terrain vehicle collision. Clinical examination shows telecanthus and periorbital ecchymosis. A fracture dislocation involving which of the following structures is most likely contributing to the telecanthus?

A) Inferior rectus muscle tendon
B) Lateral canthal tendon
C) Lateral rectus muscle tendon
D) Medial canthal tendon
E) Medial rectus muscle tendon

A

The correct response is Option D.

The medial canthal tendon is a fibrous band attached to the medial orbital wall (frontal bone and lacrimal crest). During significant trauma to the frontal and mid face region, fractures of the medial orbital wall, the nasomaxillary buttress, and the lateral nasal bones can occur and result in a naso-orbital-ethmoid fracture. With displacement of the fractures, the medial canthal tendon is also displaced laterally, resulting in telecanthus. Telecanthus is an increased distance between the medial canthi beyond the normal range for an adult. The medial canthal ligament is closely approximated to the lacrimal system and orbicularis oculi muscle.

The lateral canthal tendon is a fibrous band that attaches to the lateral orbital wall and is not involved in producing traumatic telecanthus. The lateral and medial rectus muscle tendons are the attachments of these extraocular muscles to the bony orbit. Disturbance of these muscles and/or tendons would result in strabismus.

60
Q

A 42-year-old woman is brought to the emergency department after sustaining traumatic fractures of the right orbit and zygoma in a motor vehicle collision. Physical examination shows localized edema. Which of the following indications is most likely for immediate ophthalmologic consultation?

A) Corneal abrasion
B)Diplopia
C) Eyelid ptosis
D) Hyphema
E) Subconjunctival hemorrhage

A

The correct response is Option D.

Hyphema is defined as blood within the anterior chamber of the eye. It is caused by tearing of the vessels within the iris as a result of trauma. Subconjunctival hemorrhage, on the other hand, stains the bulbar conjunctiva with blood from the site of a nearby fracture. Blood within the eye is worrisome because clotting can interfere with fluid egress from the anterior chamber, leading to the development of glaucoma. Immediate ophthalmology consultation, urgent intraocular pressure measurement, and slit-lamp examination should be performed to determine the extent of hemorrhage. Blindness, blurred vision, eye pain, globe rupture, or retrobulbar hematoma also warrant ophthalmology consultation. Patients with painful corneal abrasions have epithelial defects diagnosed by urgent (but not emergent) slit-lamp examination with topical fluorescein. Diplopia, or double vision, may be present in one or several directions of gaze following blunt trauma to the face. Neither diplopia nor traumatic ptosis warrants emergent consultation.

61
Q

A 27-year-old man is brought to the emergency department 1 hour after sustaining a knife wound to the left cheek. Physical examination shows a wound just anterior to the left ear that extends intraorally. A photograph is shown. He is able to elevate the brow, close the eyes, smile, and evert the lower lip. The laceration is irrigated thoroughly. Which of the following is the most appropriate next step in management?

A) Application of wet-to-dry dressings
B) Cannulation of Stensen duct
C) Closure of the facial wound and administration of sialogogues
D) Starch-iodine test
E) Testing of the distal branches of the facial nerve with a stimulator

A

The correct response is Option B.

After ruling out facial nerve injury, the next priority in management of a cheek laceration is to rule out injury to the parotid (Stensen) duct. Failure to repair a parotid duct laceration can result in a salivary fistula or sialocele. A small amount of methylene blue dye injected via an intravenous catheter, introduced through the ductal papilla in the mouth opposite the maxillary second molar, may be effective in identifying lacerations. Whenever possible, it is best to repair lacerations primarily using fine suture (eg, 8-0 nylon) over a stent. If the duct cannot be repaired because of extensive damage, ligation of the duct can be considered.

Wet-to-dry dressing changes and healing by secondary intention are not indicated in an early, uninfected facial wound. If a ductal injury can be ruled out, the wound should be closed primarily. It is not necessary to explore the facial nerve when the patient has clinically intact facial motor function. Such exploration risks inadvertent injury to the nerve. Anticholinergic medications, such as glycopyrrolate, can be administered to limit salivary secretion during healing and to help prevent salivary fistula or sialocele formation from an unrecognized ductal injury or glandular laceration. Sialogogues increase salivary secretion and are not effective in limiting complications. The starch-iodine test is used to assess gustatory sweating thought to occur because of inappropriate sympathetic reinnervation of the facial sweat glands after parotid surgery and is not indicated in the initial management of the patient described.

62
Q

A 24-year-old man comes to the office for follow-up examination 2 weeks after undergoing open reduction and internal fixation of a fracture of the right zygomaticomaxillary complex. The procedure was performed with intraoral and subtarsal eyelid incisions. Physical examination shows ectropion of the right lower eyelid. Ophthalmologic examination shows no vision abnormalities; the cornea is intact. Which of the following is the most appropriate management of the ectropion?

A) Coverage with a tarsoconjunctival flap
B) Lateral canthoplasty
C) Placement of an allograft to the middle lamella
D) Skin grafting to the external lamella
E) Observation with massage

A

The correct response is Option E.

Complications related to the eyelid are common following orbital fracture reconstruction. Approximately 10 to 20% of patients will require some operative revision for globe or eyelid malposition. This is because of the edema and swelling present at the initial operation. As a result, the incidence of scleral show and ectropion is also high because of eyelid retraction. The majority of the conditions described will resolve without intervention within a period of several weeks. Conservative therapy is recommended and includes tarsorrhaphy, massage, and application of ophthalmic steroid ointment or drops. Occasionally, a permanent deformity may result. Surgical treatment includes releasing the scar tissue with lateral canthal elevation. Depending on the location of the scar contracture, grafting to either the external or middle lamella may also be necessary. The incidence of corneal irritation or abrasion, as well as dry eye, may lead to earlier exploration; otherwise, conservative treatment is recommended.

63
Q

A 20-year-old man comes to the office with severe malocclusion 8 weeks after sustaining injuries during a motor vehicle collision. Physical examination shows healing lacerations, loss of sensation in the infraorbital nerve distribution on the affected side, and no orbital rim step-off deformity. CT scan shows a unilateral orbital blowout fracture with a mid face fracture. No mandibular fracture is identified. Which of the following is the most appropriate initial step in management?

A) Le Fort I osteotomy with fixation
B) Maxillomandibular fixation
C) Open reduction and internal fixation of the fracture
D) Open reduction and reconstruction of the orbital floor fracture

A

The correct response is Option A.

Because the patient is 8 weeks out from the injury, the fracture lines are immobile and a Le Fort I osteotomy and maxillomandibular fixation would be required to correct the malocclusion. Maxillomandibular fixation alone would not get the patient into normal occlusion at this late date. Open reduction and internal fixation of the zygomaticomaxillary complex fracture is not indicated at this time because the bone is solid and minimally displaced. While correction of the orbital floor fracture would be indicated if there were entrapment or increased orbital volume, this management would not address the malocclusion.

64
Q

Avulsions occur most commonly at which of the following layers of the scalp?

A) Aponeurotic layer
B) Loose areolar layer
C) Pericranium
D) Skin
E) Subcutaneous layer

A

The correct response is Option B.

The layers of the scalp can be remembered by the mnemonic SCALP: skin, subcutaneous tissue, aponeurotic layer (also called the galea), loose areolar tissue, and pericranium. The skin, subcutaneous tissue, and galea are intimately fused and move as a unit. These layers are separated from the pericranium by a layer of areolar tissue. This layer allows easy movement of the scalp and provides a space for fluid accumulation. It is through this layer that separation most easily occurs in the setting of trauma to or surgical reflection of the scalp. Emissary veins cross this space as they drain the scalp into the intracranial venous sinuses. Because of the potential for accumulation of fluid in the loose areolar tissue, an infection or abscess in this layer may lead to meningitis or septic venous sinus thrombosis. However, the incidence of such events is low.

The galeal aponeurosis is considered the strength layer of the scalp and is contiguous with the frontalis and occipitalis muscles and the temporoparietal fascia laterally. The pericranium is tightly adherent to the skull. It derives its blood supply from the diploic circulation via a multitude of perforating blood vessels and can serve as a vascularized bed for accepting skin grafts. The subcutaneous tissue contains the vessels, lymphatics, and nerves that supply the scalp.

65
Q

A 22-year-old man is brought to the emergency department after sustaining injuries to the head during a high-speed motor vehicle collision. Examination shows fluid draining from the nostrils. Beta-2 transferrin test on the fluid is positive. CT scan shows a frontal sinus fracture. The anterior and posterior tables are nondisplaced. Which of the following is the most appropriate next step?

A) Cranialization of the frontal sinus
B) Dural repair
C) Obliteration of the frontal sinus
D) Reduction and fixation of the anterior-table frontal sinus
E) Observation

A

The correct response is Option E.

A positive beta-2 transferrin test result indicates a cerebrospinal fluid (CSF) rhinorrhea. The first priority is to seal the leak. The patient should be observed before proceeding. The head of the bed should be elevated, and consideration should be given to placement of a lumbar CSF drain. If the leak does not resolve, surgical intervention should be planned.

Once the CSF rhinorrhea has resolved, operative repair of the facial fractures can begin. After the premorbid occlusion is established, the mandible fractures are plated sequentially.

Cranialization is reserved for displaced posterior table frontal sinus fractures. Dural repair is not needed at this stage as the CSF leak will probably seal on its own. Obliteration of the frontal sinus may be required if the CSF rhinorrhea fails to respond to conservative treatment.

66
Q

A 25-year-old man comes to the office for a follow-up examination because he has had intraoral drainage and pain and swelling over the right mandible for the past 24 hours. Ten days ago, he underwent maxillomandibular fixation followed by rigid internal fixation. CT scan of the head shows a soft-tissue collection along the right mandibular body; there is no evidence of osteomyelitis, and the rigid fixation is stable. Which of the following is the most appropriate management?

A) Incision and drainage
B) Incision and drainage, removal of hardware, and external fixation
C) Incision and drainage, removal of hardware, and maxillomandibular fixation
D) Incision and drainage, removal of hardware, and vascularization of fibula flap transfer
E) No additional intervention is needed

A

The correct response is Option A.

Postoperative infections are the most common complication after mandibular fracture repair, with an incidence ranging from 0.4 to 32%. Management of these infections requires adequate incision and drainage as well as intravenous antibiotics and mouth care. If the source of infection is an exposed infected tooth root, it should be removed. It is important that rigid internal fixation be maintained until the bone fracture segments have ossified because nonrigid fixation, such as external fixation or maxillomandibular fixation, can lead to worsening of infection.

Removal of the internal hardware is rarely necessary unless the hardware is actively infected and loosened because of bone involvement (osteomyelitis).

Repair with vascularized fibula flap transfer is also not indicated because there is no evidence for osteomyelitis or extensive bone loss.

67
Q

An 83-year-old man undergoes radiation therapy and surgical resection and coverage with a cranial bone graft to treat meningioma. The graft becomes infected and is removed 6 weeks after the procedure. One year later, the patient is scheduled to undergo reconstruction of the resulting 23-cm2 defect in the skull. Which of the following is a relative contraindication for the subsequent use of hydroxyapatite in this patient?

A ) Age of patient

B ) History of infection

C ) History of radiation

D ) Location of defect

E ) Size of defect

A

The correct response is Option C.

Hydroxyapatite cement is widely used in cranioplasty. In one study, the complication rate of the use of hydroxyapatite in patients who have received irradiation to the scalp was 100%. Therefore, its use is not recommended in this population.

Use of hydroxyapatite is not recommended in large, full-thickness defects in the pediatric population because it is largely nonresorbable and may be deleterious to the rapidly growing skull.

Reduction in the incidence of infection has been shown when a period of 1 year has elapsed between the initial injury or infection and the reconstruction with hydroxyapatite.

Use of hydroxyapatite in frontal areas causes twice the incidence of infection than in its use in all other areas.

Hydroxyapatite is approved by the FDA for reconstruction of bony defects up to 25 cm2 in size.

68
Q

A 41-year-old man was punched in the eye two days ago and now has numbness in his cheek and double vision. Physical examination shows paresthesias in the V2 distribution, edema of the eyelids, and proptosis. Diplopia occurs at 40 degrees of upward gaze, but there are no definite signs of entrapment. A coronal CT is shown. Which of the following is the absolute indication for repair of the orbital floor fracture in this patient?

A ) Diplopia on upward gaze

B ) Extent of orbital floor loss

C ) Medial maxillary sinus wall fracture

D ) Medial orbital wall fracture

E ) Paresthesia between the lower eyelid and upper lip

A

The correct response is Option B.

The absolute indication for repair of the orbital floor fracture in the patient described is the CT finding of loss of greater than 50% of the orbital floor. Without repair, this patient is prone to enophthalmos and long-term diplopia.

Diplopia, without evidence of entrapment, is not an absolute indication for operative repair of orbital floor fractures, especially when not within 20 to 30 degrees of primary gaze. Diplopia in extreme gazes is not particularly dysfunctional; therefore, it is only a relative indication for surgery. Definite entrapment noted on examination would be an indication for surgery, but it was not demonstrated in the patient described. Note on the CT the round shape (relative to the left side) of the inferior rectus muscle in coronal section. This shape is indicative of edema in the muscle, which is the likely cause of the diplopia in extreme upward gaze.

Concomitant medial maxillary sinus wall fracture is not an indication for operative repair of the orbital floor. Further, the patient described does not demonstrate such a fracture on the CT.

A fracture of the medial orbital wall is not present on the CT image. When present, a fracture of the medial orbital wall may make enophthalmos more likely. Enophthalmos would be an indication for surgery but also was not demonstrated.

Numbness between the lower eyelid and upper lip indicates injury to the infraorbital nerve, which is present in nearly all orbital floor fractures. It is usually a neurapraxic injury, which improves to some degree with time. It is not improved by surgery, and therefore this finding is not an indication for surgery.

69
Q

When reattaching the medial canthal ligaments during a transnasal canthal wiring procedure, which of the following is the most appropriate placement of the drill holes with respect to the lacrimal fossa?

A ) Anterior and inferior

B ) Anterior and posterior

C ) Posterior and inferior

D ) Posterior and superior

A

The correct response is Option D.

When treating congenital and traumatic deformities of the naso-orbital-ethmoid region, reconstruction and reattachment of the medial canthal tendons are often necessary. Overcorrection with this procedure is essentially impossible, and every effort should be made to prevent relapse and recurrent telecanthus. To this end, the transnasal wires containing the medial canthus should be placed through drill holes positioned posterior and superior to the posterior crest of the bony lacrimal fossa.

70
Q

A 42-year-old man comes to the office for consultation regarding the appearance of the right eye 10 months after sustaining a fracture of the right orbitozygomatic complex and multiple lacerations in a motor vehicle collision. He was unable to undergo treatment of the fractures because of other injuries. A photograph is shown. Which of the following is the most likely cause of the enophthalmos?

A ) Atrophy of globe fat

B ) Cicatrix of adnexal tissue

C ) Eyelid lacerations

D ) Increased orbital volume

A

The correct response is Option D.

Posttraumatic enophthalmos is primarily the result of increased bony orbital volume. Atrophy of globe fat and cicatrix of adnexal tissue may also contribute to enophthalmos, but minimally. Eyelid lacerations do not lead to changes in orbital volume. Clinical features of enophthalmos include narrowing of the palpebral fissure and deepening of the supratarsal fold. Enophthalmos is best assessed on submental view to evaluate globe projection. Posttraumatic enophthalmos results from displaced zygoma fractures, medial wall or floor blow-out fractures, and nasoorbito-ethmoidal fractures with disruption of the medial wall resulting in increased orbital volume.

71
Q

A 17-year-old girl is brought to the emergency department after she was hit in the left eye with a batted softball. A photograph is shown. Physical examination shows increased intraorbital pressure and decreasing visual acuity. Review of the CT scan confirms fracture of the orbital floor. Which of the following is the most appropriate immediate management?

A ) Administration of mannitol followed by exploration of the orbital floor

B ) Anticoagulation

C ) Exploration of the orbital floor and repair with a bone graft

D ) Exploration of the orbital floor and repair with synthetic material

E ) Lateral canthotomy and cantholysis

A

The correct response is Option E.

Immediate lateral canthotomy and cantholysis are the most appropriate management of the condition described. Retrobulbar hematoma or orbital hemorrhage can follow either a direct injury to the orbital contents or a fracture that involves surrounding bones. Symptoms and signs include pain, reducing visual acuity, history of trauma, periorbital/lid hematoma, chemosis, proptosis, raised intraocular pressure, and ophthalmoplegia. There is a narrow therapeutic interval of 90 minutes before permanent damage to vision may occur. Immediate exploration of the orbital floor is contraindicated when a significant retrobulbar hematoma resulting in increased intraorbital pressure is present. A delayed repair of the orbital fracture, generally within one week of the trauma, is performed if necessary.

Medical treatment of raised intraorbital pressure with mannitol or dexamethasone should be regarded as an adjunct to surgery. The first line of treatment is surgical, and lateral canthotomy at the bedside is the most effective immediate treatment for increased intraorbital pressure.

Anticoagulation is contraindicated because of suspected bleeding.

72
Q

A 33-year-old man is brought to the emergency department after sustaining injuries to the face during a snowmobile collision. Axial CT scan is shown. Which of the following is the most appropriate management?

A ) Ablation of the frontal sinus

B ) Cranialization and reconstruction of the anterior frontal sinus wall

C ) Obliteration of the frontal sinus

D ) Observation with x-ray studies monthly

E ) Reconstruction of the nasofrontal duct and anterior and posterior frontal sinus walls

A

The correct response is Option B.

A comminuted fracture of the frontal sinus is shown in the CT scan, with significant displacement of fragments involving both the anterior and posterior frontal sinus walls and the region of the nasofrontal duct. The most appropriate treatment is cranialization and reconstruction of the anterior wall to restore normal forehead contour and protect the brain. Cranialization involves removing the posterior frontal sinus wall to make the sinus part of the intracranial space and blocking the nasofrontal duct, typically with bone or a pericranial flap so that sinus mucosa is excluded from the intracranial space. The anterior frontal sinus wall is also reconstructed as part of this procedure to restore normal forehead contour and to protect the brain.

Ablation (or exenteration) involves removing the anterior frontal sinus wall and allowing the skin to collapse in on the posterior wall, if it is intact, or on the dura if the posterior wall requires removal as well (as it would in this scenario). While this may lead to a stable, healed wound in the patient described, it is not the most appropriate management as it would leave the patient with a significant deformity that would be difficult to reconstruct. It would also leave the underlying brain unprotected by bone. Ablation is appropriate only in extreme cases of acute infection that require open drainage and removal of infected bone.

Obliteration of the frontal sinus involves removing the sinus mucosa and burring the bony walls to remove mucosal invaginations, plugging the nasofrontal duct, and filling the sinus cavity with fat, muscle, bone, or alloplasts. A variation of this is osteoneogenesis, where the cavity is not filled but allowed to fill spontaneously with bone or scar over time. This would not be feasible in the patient described because of the extreme comminution and displacement of the posterior wall fracture fragments.

Observation is appropriate for minimally or nondisplaced fractures of the frontal sinus that do not involve the nasofrontal duct or do not acutely obstruct the nasofrontal duct. Regular plain x-ray studies should be obtained for several months afterward to monitor for development of a frontal sinus mucocele, which requires surgical treatment.

Reconstruction involves preserving sinus mucosa and reducing fractures of the nasofrontal duct and sinus walls. There are currently no data to support this technique, and in the patient described it could to lead to mucocele development as the nasofrontal duct became scarred and obstructed postoperatively.

73
Q

A 20-year-old man is undergoing open reduction and internal fixation of a nasoorbital-ethmoid fracture. Reconstruction of the medial canthal attachments using a transnasal approach is required. Which of the following best describes the correct placement of drill holes with respect to the posterior lacrimal crest during transnasal wiring?

A ) Anteroinferior

B ) Anterosuperior

C ) Posteroinferior

D ) Posterosuperior

A

The correct response is Option D.

When treating a severe naso-orbital-ethmoid fracture and reconstructing the medial canthal attachments, overcorrection is indicated because telecanthus is a common postoperative sequela. When performing transnasal wiring of the medial canthus, a coronal incision is often required. Identification of the lacrimal apparatus and the anterior and posterior lacrimal crests is performed. The drill holes for the transnasal wires attached to the medial canthal ligaments are placed 1 to 2 mm above and behind the lacrimal fossa or posterior and superior to the posterior lacrimal crest. Clinical confirmation of slight overcorrection is warranted because relapse is common, and telecanthus is a likely postoperative occurrence despite attempts at overcorrection.

74
Q

A 12-year-old girl is brought to the emergency department after she sustained injuries in a motor vehicle collision. Physical examination shows extensive lacerations of the right medial orbit and forehead (shown) with complete transection of the medial canthal tendon (MCT). For effective reattachment of the MCT with transnasal wiring, which of the following is the most appropriate direction of resuspension of the tendon in relation to the anterior lacrimal crest?

(A) Anterior and inferior

(B) Anterior and superior

(C) Directly horizontal

(D) Posterior and inferior

(E) Posterior and superior

A

The correct response is Option E.

The medial canthal tendon (MCT) consists of three limbs: 1) a prominent anterior limb that inserts medially on the anterior lacrimal crest, 2) a thinner posterior limb that attaches to the posterior lacrimal crest, and 3) a vertical limb of fascia that inserts on the medial orbital rim inferior to the nasal frontal suture. The resultant vectors of these attachments suggest that resuspension of the entire complex of the MCT following disruption should be posterior and superior to the anterior lacrimal crest.

75
Q

A 43-year-old woman has periodic drainage of clear fluid from the nose three months after she sustained severe trauma to the face during a motor vehicle collision. She is concerned that the drainage may be cerebrospinal fluid. Which of the following tests of nasal secretions in this patient is most specific for leakage of cerebrospinal fluid?

A ) Determination of protein content

B ) Double halo sign

C ) Glucose dipstick

D ) Measurement of beta-2 transferrin level

E ) Measurement of potassium level

A

The correct response is Option D.

Measurement of the beta-2 transferrin level is the most specific test for cerebrospinal fluid (CSF) because this molecule marker is located only in the CSF and vitreous humor of the eye. Because the test is expensive, however, it must be ordered judiciously.

Other tests for CSF are less specific and reliable. Although decreased protein and potassium levels are associated with CSF leakage, these tests are not as specific for CSF. The test for double halo sign (fluid placement on a gauze pad or filter paper and observation for double halo formation) has a sensitivity of 78% but a false positive rate of 75%. Glucose dipstick testing was the first technique used to detect CSF. It is based on the principle that the glucose content of CSF exceeds that of mucus or blood.

76
Q

A 24 year old woman is brought to the emergency department after being involved in a motor vehicle collision. Physical examination shows multiple minor abrasions of the face. Clear rhinorrhea is noted. CT of the head shows a nondisplaced fracture of the posterior table of the frontal sinus. No other serious injuries are noted. The patient is admitted to the hospital, and antibiotic therapy is initiated. Which of the following is the most appropriate next step in management?

(A) Bed rest, head elevation, and observation

(B) Cranialization of the frontal sinus

(C) Craniotomy and repair of the dural tear

(D) Lumbar puncture and drainage of spinal fluid

(E) Obliteration of the frontal sinus

A

The correct response is Option A.

The patient described has a nondisplaced fracture of the posterior table of the frontal sinus with a cerebrospinal fluid (CSF) leak and should be treated with antibiotic coverage and maneuvers to facilitate spontaneous resolution of the leak. These include bed rest and head elevation greater than 30 degrees. If the CSF leak persists for more than four days, spinal drainage is recommended. Prolonged CSF leakage for longer than seven to 10 days requires craniotomy, repair of the dural laceration, and either obliteration of the sinus or cranialization.

In treating frontal sinus fractures, the involvement of the anterior sinus wall, posterior sinus wall, dural lining, and frontonasal duct are the major determinants of the type and extent of treatment.

Isolated anterior wall fractures without depression do not require surgical treatment. However, some authors recommend treatment with antibiotics for one week. In patients with isolated depressed anterior wall fractures, surgical correction to restore aesthetic contour is indicated. If there is frontonasal duct injury, the sinus mucosa is removed and the cavity is obliterated. During sinus obliteration, the mucosa usually is removed and the sinus is curettaged with a sharp periosteal elevator or a high-speed burr. The frontonasal duct can then be occluded with a pericranial flap or a bone graft and the sinus obliterated with autogenous fat, dermal fat, muscle, cartilage, bone, pericranial graft, Surgicel, bone wax, or other material.

Posterior table fractures without displacement, CSF leakage, or frontonasal duct involvement do not require surgical intervention but do require antibiotic treatment.

Posterior table fractures without displacement, but with CSF leakage, are initially treated conservatively as described above. Should the leakage persist for longer than 10 days despite measures to resolve spontaneously, a craniotomy with dural repair is required. This procedure is usually done in conjunction with neurosurgical colleagues. Displaced posterior wall fractures require exploration with repair of any dural tears and either sinus obliteration or cranialization. Cranialization involves removal of the entire posterior table, plugging of the frontonasal duct, repair of any dural lacerations, and separation of the intracranial cavity from the aerodigestive tract. The frontal lobe is then expected to fall into and fill the previous sinus cavity.

Although the specifics of treatment of frontal sinus fractures remain controversial, the illustration below shows a simple algorithm for treating these fractures.

Long-term follow-up with annual CT is required for all frontal sinus fractures.

77
Q

A 12-year-old boy is brought to the emergency department because of double vision six hours after sustaining a blow to the eye with an elbow while jumping on a trampoline. He has had pain since the incident but has not had loss of consciousness. He had one episode of nausea and vomiting before arrival. Pulse rate is 45 bpm, respirations are 18/min, and blood pressure is 110/80 mmHg. Examination shows photophobia, periorbital ecchymosis, and restriction of extraocular motion. CT of the head shows a fracture of the orbital floor. Which of the following is the most appropriate time for surgical repair of the fracture?

(A) Emergently

(B) 1 to 3 Days

(C) 4 to 7 Days

(D) 8 to 14 Days

A

The correct response is Option A.

Observation alone is not indicated for entrapment with nausea, vomiting, and oculocardiac reflex. Recent publications in the ophthalmologic literature emphasize the importance of urgent surgical intervention to prevent ocular muscle damage, improve postoperative function, and decrease the need for additional surgery.

78
Q

A 42 year old man is brought to the emergency department 30 minutes after sustaining trauma to the face in a motor vehicle collision. Physical examination shows periorbital ecchymoses on the right, malar flattening, and enophthalmos. Neurologic examination shows numbness on the upper right lip. Radiographs show a displaced zygomaticomaxillary complex fracture. Anatomic reduction of each of the following is required in this patient EXCEPT

(A) greater wing of the sphenoid

(B) inferior orbital rim

(C) nasomaxillary buttress

(D) zygomaticofrontal suture

(E) zygomaticomaxillary buttress

A

The correct response is Option C.

Approximately 15% to 22% of patients with orbital fractures develop enophthalmos because of increased bony intraorbital volume. This is most frequently associated with a zygomaticomaxillary complex fracture that has not been reduced adequately. It is critical that an anatomic reduction be obtained to prevent the development of this difficult late complication. It is also critical that appropriate anatomic reduction of the greater wing of the sphenoid and lateral wall of the orbit is obtained after reduction of a fracture of the zygomaticomaxillary complex. Correct reduction of the inferior orbital rim, zygomaticofrontal suture, and zygomaticomaxillary buttress is also important but does not ensure the reduction of the lateral wall. The surgeon must therefore verify that the complete reduction of all structures has been performed. Reduction of the nasomaxillary buttress is not typically involved in this type of fracture pattern.

79
Q

Which of the following fractures of the facial bones is most common during childhood?

(A) Frontal sinus

(B) Le Fort

(C) Nasal

(D) Orbital

A

The correct response is Option C.

Of the options listed, nasal fractures are the most common. However, in most pediatric series they fall second to mandibular fractures. In children, mandibular fractures have been reported at 32% to 65%, nasal fractures at 12% to 45%, Le Fort fractures 2% to 8%, and orbital fractures 14% to 16%. Frontal sinus fractures are rare, except in late teenage years when pneumatization of the sinus is approaching its completion.

80
Q

A 70-year-old woman is brought to the emergency department after a fall. Examination shows periocular ecchymosis, epistaxis, and a bluish bulge of the septal mucosa. No other serious injuries are noted. CT shows fracture of the nasal septum. Which of the following is the most appropriate next step in management?

(A) Administration of intranasal oxymetazoline (Afrin)

(B) Drainage of hematomas and resection of septal cartilage

(C) Evacuation of hematomas through a direct incision

(D) Nasal packing only

(E) Needle aspiration of hematomas

A

The correct response is Option C.

Hematomas can develop between the septal mucoperichondrium and the cartilage in fractures and dislocations of the septum. Untreated septal hematomas may lead to septal perforation or fibrosis with eventual septal distortion, abscess, or complete septal necrosis with development of a saddle nose deformity. Prompt diagnosis and treatment of septal hematomas are essential to prevent such sequelae. Septal hematomas should be treated promptly with an L-shaped incision over the hematoma with thorough evacuation using suction and irrigation. This can be followed by loose repair of the incision to allow drainage and quilting sutures to prevent reaccumulation. It should be followed by internal nasal packing, systemic antibiotic coverage, and close follow €‘up to ensure absence of reaccumulation. Generally, although septal hematomas tend to be bilateral, they should not be incised on both sides because through €‘and €‘through septal perforation may occur. If the entire hematoma cannot be evacuated with a unilateral approach, the incisions on each side should be made at different levels.

Observation and administration of intranasal oxymetazoline (Afrin) spray are not appropriate treatment options because a persistent hematoma can result in significant morbidity. Resection of septal cartilage at the time of drainage should be avoided because septal perforation may occur. Needle aspiration can be used for small hematomas. However, it is not the treatment of choice and may require multiple attempts and extremely close follow €‘up to ensure resolution. Resection of septal cartilage at the time of drainage should be avoided because septal perforation may occur.

81
Q

The 27 -year -old woman shown above comes to the office for evaluation because she has had diplopia for the past 18 months. Physical examination shows orbital dystopia, enophthalmos, and malar flattening on the right. On the basis of the current findings, this patient €™s prior injury was most likely which of the following fractures?

(A) Le Fort II fracture

(B) Le Fort III fracture

(C) Orbital floor blow €‘out fracture

(D) Orbital roof fracture

(E) Orbitozygomatic complex fracture

A

The correct response is Option E.

The clinical findings of orbital dystopia, enophthalmos, and malar flattening on the patient €™s right side are typical for orbitozygomatic fractures. With the downward pull of the muscles, the zygoma rotates and the orbital volume is increased. This results in enophthalmos and possible dystopia, depending on the degree of displacement. The rotation of the zygoma causes loss of malar projection. This patient had only her orbital floor repaired with synthetic mesh. This repair was inadequate for her type of fracture, and stresses the importance of correct diagnosis and treatment in order to prevent secondary post €‘traumatic deformities.

Orbital floor or roof fractures would not result in malar flattening. Le Fort II and III fractures would cause a malocclusion, which is not present in this patient.

82
Q

A 30-year-old woman sustains a nasoethmoid fracture in a motor vehicle collision. Radiographs of the fracture site show extensive fracture comminution that extends into the point of insertion of the medial canthal tendon. Transnasal medial canthopexies are to be performed for fracture fixation. The classification of this fracture is Markowitz and Manson type 3. Which of the following is the most appropriate management?

(A) External nasal splint fixation

(B) Kirschner wire fixation

(C) Rigid plate fixation

(D) Transnasal wiring

A

The correct response is Option D.

The Markowitz and Manson system is used to classify nasoethmoid fractures according to the status of the bone fragment into which the medial canthal tendon inserts. In a type 1 fracture, there is a large central fragment. The fragment can be reduced and a plate can be applied to stabilize the nasoethmoid region. Type 2 fractures exhibit comminution of the central fragment, but the point of insertion of the medial canthal tendon remains intact. Patients with type 3 fractures have extensive comminution of the segment that extends into the point of tendon insertion. Because stabilization with a plate is not possible due to the lack of bone, transnasal canthopexy is typically required to stabilize the medial canthal tendon.

Nasoethmoid fractures can be classified further according to whether they are unilateral or bilateral and their extension into other areas of the facial skeleton (see the figures below).

83
Q

A 24-year-old man is brought to the emergency department one hour after he sustained injuries to the face and head in a motor vehicle collision. Physical examination shows a 5 x 1.5-cm laceration of the forehead, ecchymosis over the glabellar region, and watery nasal discharge. CT of the head shows a fracture of the frontal sinus. Which of the following findings on analysis of the nasal discharge confirms cerebrospinal rhinorrhea?

(A) Alpha fetoprotein

(B) Beta-2 transferrin

(C) Glucose

(D) Halo sign

(E) Potassium

A

The correct response is Option B.

The most common clinical finding in a frontal sinus fracture is a laceration of the forehead. Any hematoma or ecchymosis in the glabellar region may indicate serious injury. Part of the evaluation for a frontal sinus fracture includes examining the nose for cerebrospinal fluid (CSF) leak. There are multiple modalities to evaluate for the presence of CSF leak, including the halo sign, which demonstrates a halo effect on filter paper from the draining spinal fluid, evaluation of the fluid for glucose, total protein, or electrolytes. Although all of these are possible, they lack in sensitivity and specificity. The best method of evaluating for CSF rhinorrhea is beta-2 transferrin. Beta-2 transferrin is a protein produced by neuroaminidase activity in the brain, which is uniquely found in the CSF and perilymph of the central nervous system. Beta-2 transferrin is absent in other body secretions. Therefore, its detection is invaluable in diagnosing a CSF leak. Rhinorrhea containing beta-2 transferrin does not, however, demonstrate the location of the CSF leak. This is confirmed either operatively or radiologically using high-resolution CT and magnetic resonance cisternography. Alpha fetoprotein is used as a serum marker in hepatocellular carcinoma and nonseminomatous germ cell tumors but is not used to identify CSF leaks.

84
Q

A 37-year-old woman is brought to the emergency department two hours after she sustained injuries to the face in a motor vehicle collision. Radiographs show an orbital zygomatic fracture on the left side. On ophthalmologic examination, the left pupil fails to constrict when direct light is shined in the eye; consensual pupillary constriction is normal. Color perception is diminished in the left eye. Findings in the right eye are normal. Which of the following is the most likely cause of these findings?

(A) Detachment of the retina

(B) Extension of the fracture through the optic canal

(C) Impingement of bone fragments on the optic (II) nerve

(D) Shear force injury to the optic (II) nerve

(E) Thrombosis of the retinal artery

A

The correct response is Option D.

Physical examination of the patient with an orbital zygomatic fracture includes an ophthalmologic assessment. Along with evaluation for motility disturbances of the globe and diplopia, a direct visual assessment is performed. The finding of traumatic optic neuropathy has been reported in 2% to 5% of patients with severe facial trauma. In its most severe form, traumatic optic neuropathy results in visual loss; however, findings may be more subtle, such as diminished color perception. It is also possible to find an afferent pupillary defect in the affected eye (if the pupil fails to constrict with direct light stimulation but constricts normally in a consensual response when light is directed to the contralateral eye). Injury to the optic (II) nerve can result from either mechanical or ischemic insults. The most likely mechanism of injury is shear force to the optic (II) nerve. The dural sheath is firmly attached to the optic nerve at its entrance into the optic foramen. Rapid deceleration injuries of the head can generate forces that are concentrated at the optic foramen. An optic nerve injury can also result from fracture fragments directly damaging the optic nerve or globe, thrombosis of the retinal artery, or retinal detachment; however, these are less common than injury resulting from shear force.

85
Q

A 38 -year-old man sustains panfacial fractures in a motor vehicle collision. During surgical reduction, which of the following structures is most appropriate to restore the transverse dimension of the facial skeleton?

(A) Condyle and posterior mandibular ramus

(B) Mandibular arch

(C) Nasomaxillary buttress

(D) Pterygomaxillary buttress

(E) Zygomaticomaxillary buttress

A

The correct response is Option B.

The buttresses of the face allow more precise reduction of facial fractures in both the transverse and vertical dimensions. The vertical buttresses include the nasomaxillary, zygomaticomaxillary, and pterygomaxillary buttresses. The condyle and posterior mandibular ramus make up yet another buttress establishing posterior facial height. The horizontal buttresses, also known as anterior €‘posterior buttresses, allow precise restoration of facial depth. They include the frontal, zygomatic, maxillary, and mandibular buttresses. The mandibular buttress is composed of the mandibular arch.

Of the options listed, the mandibular arch is the only structure that defines the horizontal buttress of the face.

86
Q

A 25 year old man is brought to the emergency department 30 minutes after sustaining severe trauma to the head and face during a motor vehicle collision. Examination shows clear-fluid rhinorrhea, indicating leakage of cerebrospinal fluid. This patient has most likely sustained an injury to which of the following structures?

(A) Anterior ethmoidal air cells

(B) Cribriform plate

(C) Frontonasal duct

(D) Orbital portion of frontal bone

(E) Superior nasal concha

A

The correct response is Option B.

The cribriform plate (the horizontal component of the ethmoid bone) has many foramina that act as passageways for olfactory (I) nerves and is in intimate contact with the meninges. Therefore, injury to the cribriform plate may cause tearing of the meninges, allowing leakage of cerebrospinal fluid via intact or fractured foramina. The other components of the ethmoid bone are the vertical portion and the lateral masses.

Because of their location, the other structures listed are not likely to be involved with leakage of cerebrospinal fluid. The lateral masses of the ethmoid bone extend from the periphery of the cribriform plate, contain the anterior ethmoid air cells, and articulate with the orbital portion of the frontal bone. The crista galli is a midline prominence that serves as a point of attachment for intracranial soft tissue and that centers the cribriform plate. The frontonasal duct communicates the frontal sinus of the nasal cavity. The superior nasal conchae are bony prominences located on the medial side of each lateral mass and facing the nasal cavity.

87
Q

A 1-month-old infant is brought to the office by her parents for evaluation of a skull deformity. CT scan of the head is shown above. Which of the following cranial dysmorphologies is the most likely diagnosis?
(A) Dolichocephaly
(B) Plagiocephaly
(C) Scaphocephaly
(D) Trigonocephaly
(E) Turricephaly

A

The correct response is Option D.

The CT scan demonstrates an infant with a narrowed forehead and decreased bitemporal distance. There is also evidence of orbital hypotelorism and an obliterated or fused metopic suture. These findings are consistent with trigonocephaly.

Scaphocephaly is a cranial shape excessive in the anterior-posterior dimension and narrow in the bitemporal dimension. This results in a long and narrow (boat-like) shape of the head. This results from premature fusion of the sagittal suture.

Dolichocephaly is another term for scaphocephaly.

Plagiocephaly or “twisted head” is used to describe anterior or posterior deformities. Plagiocephaly is classified as anterior or posterior as well as synostotic and nonsynostotic or deformational. Anterior synostotic plagiocephaly refers to coronal craniosynostosis and posterior synostotic plagiocephaly refers to lambdoid craniosynostosis. Anterior or posterior nonsynostotic plagiocephaly refers to deformational plagiocephaly or skull molding from a persistent sleep position.

Turricephaly or oxycephaly is used to describe vertically tall head shapes that are usually associated with the brachycephaly (short in the anterior-posterior dimension) of bicoronal syndromic craniosynostosis.

88
Q

A 20-year-old man who sustained a naso-orbital-ethmoid fracture undergoes transnasal wire canthopexy. Which of the following vectors, relative to the position of the insertion of the medial canthal tendon, is most appropriate for placement of the primary wire?

(A) Anterior and inferior

(B) Anterior and superior

(C) Posterior and inferior

(D) Posterior and superior

A

The correct response is Option D.

Transnasal key wire must be positioned posterior and superior to the insertion of the medial canthal tendons, behind and above the lacrimal fossa (see diagram below). The fractures are exposed via coronal approach and may require temporary removal of nasal bones for complete access. The vector provided with this location of wire positioning allows optimal relationship between the eyelid tissue and the globes. The canthal tendon is rarely completely detached from the bone and usually has fibers to the anterior and posterior lacrimal crests. However, in cases of tendon avulsion, the same principle applies, so the tendon must be transfixed posterior and superior to the lacrimal fossa. The bony intercanthal distance should be between 16 and 23 mm.

Additional pull €‘out transnasal soft-tissue bolsters should be applied anteriorly to assist with contouring of soft tissue against bone.

89
Q

The CT scan shown demonstrates a fracture of which of the following bones?
(A) Ethmoid
(B) Maxillary
(C) Palatine
(D) Parietal
(E) Sphenoid

A

The correct response is Option E.

In the CT scan, the lateral orbital wall is fractured. The lateral orbital wall is made up of the zygoma and the greater wing of the sphenoid. The medial orbital wall is made up of the ethmoid and palatine bones, and no fracture of these is demonstrated. The parietal bone largely is a cranial bone, which is not fractured in this CT scan. The maxillary bone, making up the buttresses and walls of the maxillary sinus, a portion of the orbital floor, and the medial aspect of the inferior orbital rim, is not fractured in this CT scan.

90
Q

A 7-year-old boy is brought to the emergency department after sustaining trauma to the face. He has nausea and vomiting but is alert and oriented to time and place. Pulse rate is 48/min. Physical examination shows right periorbital ecchymosis, diplopia, and limited vertical gaze. Visual acuity is within normal limits. Which of the following is the most likely finding on CT scan of the head and craniofacial skeleton?

(A) Compression of the optic nerve

(B) Displaced fracture of the zygoma

(C) Large (greater than 3 cm) displaced fracture of the orbital floor

(D) Minimally displaced fracture of the orbital floor

A

The correct response is Option D.

Based on the patient €™s age, signs, and symptoms, a CT scan is most likely to show a minimally displaced fracture of the orbital floor, suggesting a trapdoor fracture of the orbital floor. This type of fracture allows herniation of the orbital contents, which are then entrapped. It most commonly occurs in children, possibly because of the relative elasticity of their bones, which allows them to snap back into position after tissue herniates through the fracture. As in this patient, a trapdoor fracture of the orbital floor can cause an oculocardiac reflex, which produces bradycardia, nausea, and syncope.

A CT scan is not likely to show compression of the optic nerve because the patient does not have a relative afferent pupillary defect. A CT scan also is unlikely to show a displaced fracture of the zygoma because this type of fracture usually occurs in older patients. This study should not disclose a large displaced fracture of the orbital floor because this type of fracture does not typically cause oculocardiac reflex. Because the patient €™s mental status is intact, a CT scan is not likely to show a subarachnoid hemorrhage.

In a trapdoor fracture of the orbital floor, the muscles most likely to be entrapped are the inferior rectus and inferior oblique muscles. If entrapment restricts ocular movement, immediate surgery is recommended to prevent irreversible ischemia and fibrosis of the entrapped contents.

91
Q

A 25-year-old man comes to the office because he has numbness of the tip of the nose six weeks after he sustained a naso-orbital-ethmoid fracture during a motor vehicle collision. The most likely cause of the numbness is damage to which of the following nerves?

(A) Anterior ethmoidal

(B) Infraorbital

(C) Infratrochlear

(D) Nasopalatine

(E) Pterygopalatine

A

The correct response is Option A.

The anterior ethmoidal nerve innervates the nasal tip. The infratrochlear and infraorbital nerves innervate the sidewalls and dorsum. The lateral branch of the pterygopalatine nerve supplies the upper and middle turbinates, whereas the medial branch supplies the septum.

The nasopalatine nerve, which is the terminal branch of the pterygopalatine nerve, innervates the maxillary incisor teeth, gingiva, and palate.

92
Q

In a patient with fracture of the frontal sinus, laboratory analysis of which of the following is most reliable to confirm cerebrospinal rhinorrhea?

(A) Albumin

(B) Beta-2 transferrin

(C) Glucose

(D) Glutamine

(E) Lactic acid

A

The correct response is Option B.

Initially described for the detection of cerebrospinal fluid (CSF) in 1979, beta-2 transferrin is currently accepted as the most reliable method of laboratory testing in determining the presence of CSF in rhinorrhea and otorrhea. It has been reported to have a sensitivity near 100% and a specificity near 95%. Beta-2 transferrin is a carbohydrate €‘free (desialylated) isoform of transferrin that is almost exclusively found in CSF. It is also found in perilymph in the cochlea and the aqueous and vitreous humor of the eye, but in lower concentrations. It is not present in blood, nasal mucus, tears, or mucosal discharge. Its presence is identified using immunofixation electrophoreses. Glucose testing of rhinorrhea or otorrhea was commonly used in the past, but interpretation of the studies was confounded by contamination of glucose containing fluid (tears, nasal mucus, blood in nasal mucus) or low CSF glucose levels (as seen in meningitis). Glucose testing of CSF has been found to have an unacceptably high rate of false-positive results.

93
Q

Cranialization is indicated for patients with which of the following conditions of the frontal sinus?

(A) Fracture of the anterior sinus wall

(B) Fracture of the nasofrontal duct

(C) Fracture of the posterior sinus wall

(D) Fracture of the sinus floor into orbital roof

(E) Post-traumatic frontal sinus mucocele

A

The correct response is Option C.

Although cranialization is indicated for fractures of the posterior sinus wall, not all frontal sinus fractures involving the posterior wall require cranialization. Exceptions would be noted in cases of undisplaced fracture in which the dura is deemed intact by clinical observation and confirmatory testing. Even in patients with leakage of cerebrospinal fluid (CSF), many surgeons will allow up to 10 days for the CSF leak to resolve on its own before resorting to cranialization, as long as all fractures are undisplaced and the nasofrontal duct is patent. In addition, cases of posterior wall fracture with obstructed nasofrontal ducts can be addressed with sinus obliteration and not cranialization, as long as there is no brain injury and little or no comminution of the posterior wall. If there are other potential sources of a CSF leak, sorting out the cause can be done preoperatively with a metrizamide CT scan. The cranialization procedure consists of obliterating the nasofrontal duct and removing the sinus mucosa and posterior wall of the sinus, effectively incorporating the sinus space into the cranial cavity. This prevents future meningitis and brain abscess by sterilizing this connection.

Management of an anterior table fracture with an intact posterior table and an intact frontonasal duct should be addressed by plating the anterior table to restore forehead contour.

If the nasofrontal duct is obstructed, then obliteration of the frontal sinus is indicated.

When the sinus floor is fractured medial to the supraorbital foramen, there is a good chance that the nasofrontal duct is injured, in which case sinus obliteration is indicated. If it is not injured, then plating is indicated to restore the orbital roof if fracture lines are displaced.

Post-traumatic mucoceles of the frontal sinus are managed by sinus obliteration. The exception is, however, that cranialization could be indicated in a few instances where the mucocele has become extremely expanded and destructive on the posterior table.

94
Q

A 7-year-old boy is being evaluated after sustaining facial injuries when he fell while climbing playground equipment. Radiographs show a fracture of the orbit. Which of the following additional findings best supports urgent surgical repair in this patient?
(A) Acute enophthalmos
(B) Corneal abrasion
(C) Diplopia on upward gaze
(D) Entrapment of the rectus muscle
(E) Hyphema

A

The correct response is Option D.

Surgical repair of orbital fractures can be classified as emergent, urgent, or delayed. Most orbital fractures can be safely and effectively repaired within two weeks. This allows periorbital edema to resolve and makes the dissection easier. Indications for repairing orbital fractures include persistent diplopia in central gaze, early enophthalmos or vertical dystopia, and large fracture size, which may predispose later development of enophthalmos. Such fractures are repaired with some urgency within two weeks of the injury.

In the pediatric population, there is a subset of orbital fractures which require emergent repair, i.e., repair ideally within the first 24 hours after injury. A trapdoor fracture refers to an orbital floor fracture that, because of the elastic recoil of cartilaginous bone, traps orbital contents and the inferior rectus muscle within the maxillary sinus. Often, the recoiled floor appears uninjured. CT scan, however, will demonstrate the incarcerated muscle. This should be considered an ischemic injury, as demonstrated in the CT scan. Improved muscle function has been demonstrated with earlier surgical correction.

Hyphema represents blood from hemorrhage in the anterior chamber of the eye. Ocular injury is a contraindication to early surgical intervention. Orbital manipulation increases the risks of secondary bleed into the anterior chamber and the development of acute closed-angle glaucoma.

95
Q

A 36-year-old man has fever and headache six months after sustaining a frontal sinus fracture involving the frontonasal duct in a motor vehicle collision. At the time of injury, he underwent open reduction and internal fixation of the anterior wall of the sinus using microplates and obliteration of the sinus with fat grafts. Current CT scan shows a mucocele. Which of the following is the most likely cause of this finding?
(A) Fat necrosis
(B) Hardware failure
(C) Osteomyelitis
(D) Retained sinus mucosa
(E) Undiagnosed fracture of the posterior wall

A

The correct response is Option D.

Retained sinus mucosa is most likely to cause a mucocele or mucopyocele. Although they typically occur in patients with untreated fractures of the frontal sinus, mucoceles can also develop if all of the mucosal lining is not removed during obliteration of the sinus. A mucocele is generally sterile and filled with secretions; if it becomes infected by bacteria, it is referred to as a mucopyocele.

Because a mucocele will continue to expand and cause pressure symptoms if left untreated, complete removal of all mucosal elements using a diamond-cut rotational burr is advocated. In addition, the duct and sinus should be obliterated using free cancellous bone grafts or vascularized soft-tissue flaps (i.e., a galeal flap), and the sinus wall should then be reconstructed. Any infection should be treated initially and allowed to resolve before the cavity is filled with graft material.

Osteomyelitis may result from extension of an infected external wound, mobilization of bone fragments, or devascularization of soft tissue. This condition may also occur in conjunction with a bone-eroding mucopyocele, but would by itself not lead to development of mucocele.

Fractures of the posterior wall of the frontal sinus can lead to meningeal tears and subsequent cerebrospinal fluid leak. The fracture fragments may entrap the torn edges of the mucosa from within the sinus, leading to meningitis or development of an abscess resulting from transmission of contaminated sinus secretions.

Cancellous bone and vascularized soft-tissue flaps are the preferred fill materials for sinus obliteration procedures. Free fat and muscle grafts are associated with necrosis and potential resorption. Use of hydroxyapatite cement has produced results inferior to cancellous bone. Likewise, devascularized soft-tissue grafts are not recommended.

96
Q

A 31-year-old man undergoes open reduction and internal fixation of a naso-orbital-ethmoid fracture. During the procedure, avulsion of the right medial canthal tendon is noted. Which of the following is the most appropriate management?
(A) No intervention is needed
(B) Application of a long nasal splint
(C) Transnasal canthopexy
(D) Placement of a lacrimal stent
(E) Dacryocystorhinostomy

A

The correct response is Option C.

During open reduction and internal fixation of the naso-orbital-ethmoid fracture, care must be taken to ensure that the medial canthal tendon is not detached from the bone fragment. If the bone fragment is free but the canthal tendon is intact, the bone fragment can be fixed using transnasal fixation such as a transnasal wire. If the canthal tendon is partially detached, simple observation is likely to lead to complete detachment and telecanthus, which is difficult to treat postoperatively. Therefore, the most appropriate management of partial detachment of the medial canthal tendon is transnasal canthopexy with or without totally detaching the medial canthal tendon. Application of a long nasal splint is unlikely to keep the tendon adherent to the frontal process of the maxilla. Lacrimal stenting or dacryocystorhinostomy may be indicated for other reasons such as lacrimal obstruction; however, this is not necessary for the management of a detached medial canthal tendon.

97
Q

A 48-year-old man is brought to the emergency department one hour after he sustained injuries to the face in a motor vehicle collision. Radiographs show an orbital zygomatic fracture. Which of the following is the single most important landmark for proper alignment of this fracture?
(A) Infraorbital rim
(B) Zygomaticofrontal buttress
(C) Zygomaticomaxillary buttress
(D) Zygomaticosphenoid articulation
(E) Zygomaticotemporal buttress

A

The correct response is Option D.

Seven bones comprise the orbit: ethmoid, frontal, lacrimal, maxilla, palatine, and greater and lesser wings of the sphenoid. The orbital wall is formed primarily by the orbital surface of the zygomatic bone and the greater wing of the sphenoid bone. The sphenoid portion of the lateral orbit is separated from the roof of the orbit by the superior orbital fissure and from the floor by the inferior orbital fissure. A classic orbital zygomatic fracture requires a fracture through the lateral orbital wall, which represents articulation of the zygoma with the greater wing of the sphenoid. Because this is a broad articulation, it provides a good location to assess the degree of displacement and malposition of the fracture fragment. Anatomic reduction of the greater wing of the sphenoid and lateral wall of the orbit is critical to proper reduction for an orbital zygomatic fracture. Although the infraorbital rim, zygomaticofrontal buttress, zygomaticomaxillary buttress, and zygomaticotemporal buttress are all important landmarks and need to be assessed for degree of fracture comminution, it is possible to reduce these segments while the zygoma and sphenoid articulation is out of alignment.

98
Q

A 40-year-old man has swelling of the face and bleeding from the nose after he was kicked by a horse. Physical examination shows fracture of the frontal sinus. CT scan of the head shows pneumocephalus and displacement of the anterior and posterior walls of the frontal sinus secondary to comminuted fractures of both sinuses. Which of the following is the most appropriate management?
(A) Ablation of the frontal sinus
(B) Cranialization of the frontal sinus
(C) Enlargement of the frontonasal duct
(D) Exenteration of the frontal sinus
(E) Obliteration of the frontal sinus

A

The correct response is Option B.

Cranialization of the frontal sinus is the most appropriate management of a patient with significant displacement of both the anterior and posterior tables of the sinus. This procedure allows close inspection of the dura for possible tear as well as dealing with possible injury of the nasofrontal duct. Craniotomy via a bicoronal approach is used to expose the fracture, and then the posterior table and sinus mucosa are removed. The nasofrontal duct is occluded with a pericranial flap to disrupt the communication of the duct.

Ablation involves removing the frontal sinus in its entirety and is no longer done because of the secondary cosmetic defects.

Enlargement of the frontonasal duct via stenting or dilation to ensure adequate drainage of the sinus is used in isolated fractures around the duct and would not address the problem of the posterior and anterior tables.

Exenteration is removal of the anterior table of the sinus alone and is used in isolated cases of anterior wall fracture when reconstruction is not an option at the time, i.e., due to infection.

Obliteration of the sinus is done by removing all the mucosa within the frontal sinus and allowing the nasofrontal duct to occlude. Frequently, graft material is used to fill this space. This technique is used in patients in whom the patency of the nasofrontal duct is questioned.

99
Q

A 35-year-old man has swelling and tenderness of the nose and deviation of the nose to the left after being accidentally struck in the face while playing squash. Intranasal examination shows a localized purple mass on the left side of the septum. A CT scan is shown on page 4. Which of the following is the most appropriate initial management?

(A) Immediate closed reduction and placement of an external splint
(B) Immediate open reduction of the nasal fractures
(C) Immediate incision and drainage of the septal mass
(D) Immediate submucous resection of the septal fracture

A

The correct response is Option C.

The most appropriate initial step in the management of this patient is immediate incision and drainage of the septal hematoma. If left untreated, septal hematomas cause fibrosis and narrowing of the nasal passages, distortion of the septum, and/or formation of an abscess. They can also cause pressure necrosis of the septum, leading to septal perforation and eventually to complete necrosis with formation of a saddle-nose deformity.

Fracture management is undertaken after the hematoma has been evacuated. In patients who have significant swelling obscuring the nasal structure, fracture reduction is delayed until the swelling resolves, typically within five to 10 days. If swelling is minimal, then reduction can be performed immediately after evacuation.

Although closed reduction is appropriate in most patients with nasal fractures, open reduction may be necessary to obtain anatomic reduction. If lacerations are present, they may be used as incisions for surgical reduction. However, it should be noted that open techniques have been associated with a small incidence of necrosis of the traumatized nasal mucosa. In addition, unsatisfactory results following surgery are likely to lead to severe injury of the nasal septum that may be difficult to correct. Formal rhinoplasty may be performed at a later date to resolve breathing difficulties or improve the aesthetic appearance of the nose.

100
Q

A 12-year-old boy has significant malocclusion after sustaining facial injuries in a motor vehicle collision. A three-dimension CT scan shows a fracture of the condyle on the left and a displaced symphyseal fracture. Which of the following is the most appropriate management?

(A) Maxillomandibular fixation for four weeks
(B) Maxillomandibular fixation for 10 to 14 days
(C) Open reduction and fixation of both fractures
(D) Open reduction and fixation of the symphyseal fracture followed by maxillomandibular fixation for two weeks
(E) Rest and initiation of a soft diet

A

The correct response is Option D.

In this child who has significant malocclusion resulting from fractures of the condyle and symphysis, the most appropriate management is open reduction and internal fixation of the symphyseal fracture, followed by a short course (10 to 14 days) of maxillomandibular fixation. Children who sustain trauma to the face frequently have multiple injuries, and evaluation with CT scans is necessary. Open reduction is the treatment of choice for symphyseal fractures. However, rigid fixation with metallic plates and screws remains controversial in children, and some surgeons advocate the removal of these plates and screws after the fracture has healed. Others recommend absorbable plates and screws combined with a short course of maxillomandibular fixation. When the maxillomandibular fixation is removed, active physical therapy should be initiated because children are at increased risk for development of ankylosis of the temporomandibular joint.

Maxillomandibular fixation presents a challenge to plastic surgeons when performed in children who are at the age of mixed dentition. This is defined as the presence of both deciduous (primary) and permanent (secondary) teeth within the oral cavity simultaneously; it typically occurs between ages 8 and 10 years. In these children, it is imperative for the surgeon to be aware of the potential for injury to the tooth buds when applying rigid fixation for management of facial fractures.

Rest and initiation of a soft diet are inappropriate because of the risk for significant malocclusion. Instead, this course of treatment, combined with physical therapy, is recommended for children who have facial fractures not associated with malocclusion.
Maxillomandibular fixation is not indicated for more than 10 to 14 days because of the speed of healing in young children and the risk for ankylosis occurring with prolonged immobilization, as mentioned above.

In children, the mandibular condyle is immature, highly vascular, and covered with a thin sheath of periosteum. It is commonly described as a “vascular sponge.” As a result, open reduction of pediatric condylar fractures is discouraged.

101
Q

A 22-year-old woman comes to the office two months after sustaining injuries to the face during a motor vehicle collision. Because of other injuries, repair of the facial injuries was delayed. Physical examination shows enophthalmos and increased width of the midface. This patient most likely has which of the following facial fractures?

(A) Le Fort I
(B) Le Fort II
(C) Le Fort III
(D) Mandibular subcondylar
(E) Zygomatic complex

A

The correct response is Option E.

This patient is most likely to have a fracture of the zygomatic complex because the zygomatic arch is the main contributor to width of the midface. This width is reciprocally related to projection of the malar eminence. If the zygoma is inadequately reduced, projection of the malar eminence typically decreases and facial width typically increases secondary to lateral displacement of the zygomatic arch.

Le Fort I, II, and III fractures can change the vertical height of the face and projection of malar eminences but do not routinely affect facial width. A mandibular subcondylar fracture can also change the height of the face but not its width.

102
Q

A 27-year-old man has malocclusion and tenderness around the orbits and bridge of the nose after sustaining facial injuries in a motor vehicle collision. A photograph and CT scan are shown above. The patient is to undergo open reduction and internal fixation of the fractures.

Which of the following is the most appropriate management of the lacrimal system?

(A) Observation
(B) Exploration of the lacrimal duct
(C) Placement of a silicone stent
(D) Immediate dacryocystorhinostomy
(E) Primary repair followed by dacryocystorhinostomy in three months

A

The correct response is Option A.

In this patient who has sustained a naso-orbitoethmoid fracture, the most appropriate management of the lacrimal system is observation. The incidence of injury to the nasolacrimal duct in patients who undergo open reduction and internal fixation of naso-orbitoethmoid fractures but have no overlying lacerations is fairly low. Although swelling or fracture may contribute to blockage of the duct at the time of the initial injury, approximately 90% of patients will experience improvement of symptoms with resolution of the swelling and reduction of the fracture.

Exploration and/or manipulation of the duct are not recommended if there is no obvious injury to the duct. These procedures may only result in further damage because of the edema and friability of the tissues. Immediate dacryocystorhinostomy is not warranted for the same reason.

Patients who have persistent epiphora after resolution of swelling should undergo further evaluation. Dacryocystography can be performed for assessment of possible nasolacrimal duct occlusion. If occlusion is present, dacryocystorhinostomy is indicated.

103
Q

The CT scans shown above are from a 25-year-old man who sustained facial injuries in a motor vehicle collision. In this patient, rigid fixation at which of the following points is most likely to result in stable reduction of the fractures?

(A) Nasomaxillary buttress, inferior orbital rim, and zygomaticomaxillary buttress
(B) Zygomatic arch, central fragment, and nasomaxillary buttress
(C) Zygomatic arch, inferior orbital rim, and zygomaticomaxillary buttress
(D) Zygomaticofrontal suture, inferior orbital rim, and zygomaticomaxillary buttress
(E) Zygomaticofrontal suture, orbital floor, and zygomaticomaxillary buttress

A

The correct response is Option D.

This patient has sustained a fracture of the zygomaticomaxillary complex, also known as a zygoma fracture. To ensure stable reduction of the fracture, rigid fixation is applied at the zygomaticofrontal suture, inferior orbital rim, and zygomaticomaxillary buttress. Partial relapse may occur without this three-point rigid fixation.

Central fragments are associated with naso-orbitoethmoid fractures, not zygomaticomaxillary complex fractures. This fragment is comprised of the ascending frontal process of the maxilla and the descending internal angular process of the frontal bone. It provides the bony support for the medial canthus.

The nasomaxillary buttress is comprised of the piriform rim and is osteotomized routinely during elective Le Fort I maxillary advancement procedures. This structure is typically involved in midface fractures and not in fractures of the zygomaticomaxillary complex.

The orbital floor is often involved in zygomaticomaxillary complex fractures but not in simple fractures of the zygoma. Therefore, open reduction and internal fixation of the orbital floor may not be necessary to ensure correction of the zygomaticomaxillary fracture.

The zygomatic arch is contained within a periosteal sleeve and is often reduced and fixed adequately with adequate reduction and fixation of the zygomaticomaxillary complex. Open reduction and internal fixation through a coronal approach may be required if the zygomatic arch is comminuted significantly, but this is rare.

104
Q

A 23-year-old man has ectropion and shortening of the lower lid of the left eye two months after undergoing repair of an isolated orbital floor blowout fracture with a cranial bone graft via a subtarsal approach. Physical examination shows 2 mm of scleral show and 1 mm of lagophthalmos. Examination of the eyes shows normal Bell phenomenon. Which of the following is the most appropriate initial management?

(A) Kuhnt-Szymanowski procedure
(B) Lateral canthoplasty
(C) Massage of the lower lid
(D) Nasal septal cartilage grafting
(E) Release of scar tissue and application of a Frost traction suture

A

The correct response is Option C.

The initial management of ectropion and lower eyelid shortening is massage. This patient has minimal scleral show, lagophthalmos, and normal Bell phenomenon, which suggest minimal risk to the cornea. In many cases, scleral show and lagophthalmos improve spontaneously with maturation of scar tissue. Six months of conservative management should be allowed before surgical intervention is undertaken.

If ectropion and lid shortening persist for more than six months after the original surgical procedure or if the cornea is at risk for desiccation and injury, surgical correction should be performed. Corrective options include Kuhnt-Szymanowski procedure (horizontal shortening of the lower lid), lateral canthoplasty, release of scar tissue and application of a Frost traction suture, and nasal septal cartilage grafting to support the posterior lamella.

105
Q

The firstborn child of a Caucasian couple with no abnormalities has bilateral cleft lip and palate. Which of the following percentages best represents the possibility that this couple=s next child will have cleft lip, with or without cleft palate?

(A) 2%
(B) 4%
(C) 8%
(D) 16%
(E) 32%

A

The correct response is Option C.

The correct answer for the sibling of a child with bilateral cleft lip and palate is between 5.7% and 8.0%, depending on the reference. Regardless, the risk decreases if there is no cleft palate in the proband and/or if the condition is unilateral. The incidence of cleft lip with or without cleft palate for the sibling of a child with bilateral cleft lip but no cleft palate is 6.7%, for the sibling of a child with unilateral cleft lip and palate is 4.9%, and for the sibling of a child with unilateral cleft lip but no cleft palate is 4.0%. If there are two affected children with cleft lip with or without cleft palate, the risk for a third child would be at least 9%.

The overall risk of cleft lip with or without cleft palate in a Caucasian population is 1:1000. In a first-degree relative, that risk is multiplied by 40; in a second-degree relative, the risk is multiplied by 7; and in a third-degree relative, the risk is multiplied by 3. However, the risk is increased when more than one relative is affected, if the family member has a severe form of the disorder, if the family member is of the sex least likely to be affected, or if the parents are consanguineous. For one affected parent, the risk of having one child with cleft lip with or without cleft palate is 4%, and the risk for a second child increases to 17%.

106
Q

A 26-year-old woman comes to the office for consultation one year after undergoing open reduction and internal fixation of a fracture of the left zygomaticomaxillary complex. During that procedure, a titanium plate was placed for reconstruction of the orbital floor defect. Examination shows enophthalmos and malar asymmetry. Which of the following is the most likely cause of enophthalmos in this patient?

(A) Atrophy of the intraorbital fat contents
(B) Fibrosis of the extraocular muscles
(C) Herniation of the orbital contents into the maxillary sinus
(D) Inadequate reduction of the fracture
(E) Inadequate resuspension of the soft tissue of the cheek

A

The correct response is Option D.

The major cause of persistent enophthalmos after surgery is inadequate reduction of the fracture fragments. The key to reduction of a zygomaticomaxillary complex fracture is accurate reduction of the interface between the zygomatic and sphenoid bones. This interface, which comprises the lateral wall of the orbit, is a broad interface between bones as opposed to the narrower interfaces at the orbital rim and zygomatic frontal suture. At these locations, an apparent reduction can actually be rotated about the vertical axis and misaligned at the lateral orbit. This misalignment increases the orbital volume, which allows the eye to sink back into the orbit, causing enophthalmos.

Atrophy of intraorbital fat, fibrosis of the extraocular muscles, and herniation of the orbital contents can also cause a mismatch between the volume of the orbit and its contents. However, these options do not account for the malar asymmetry. Fibrosis of the extraocular muscles can be ruled out by the forward traction and forced duction tests, which also rule out entrapment of the extraocular muscles.

Inadequate resuspension of the soft tissue of the cheek might account for the malar asymmetry but would not contribute to enophthalmos. The cheek tissue is external to the orbit, and the enophthalmos results from a discrepancy between the volume of the bony orbit and its contents.

107
Q

An 18-year-old woman sustains an isolated depressed fracture of the zygomatic arch when she is struck on the left cheek with a baseball. Fracture reduction is planned using the temporal (Gillies) approach. During this procedure, an elevating device should be inserted into which of the following tissue planes?

(A) Between the skin and the superficial temporal fascia
(B) Between the superficial temporal fascia and the deep temporal fascia
(C) Between the deep temporal fascia and the temporalis muscle
(D) Between the temporalis and lateral pterygoid muscles
(E) Deep to the masseter muscle

A

The correct response is Option C.

The temporal (Gillies) approach to reduction of a zygomatic arch fracture involves creating an incision in the temporal region and inserting an elevating device between the deep temporal fascia and the temporalis muscle. The strong layer of fascia attached to the upper border of the zygomatic arch guides the instrument into a position immediately beneath the malar eminence, allowing the surgeon to reposition the depressed fracture fragments both laterally and outward.

If the elevating device is passed through the subcutaneous plane (between the skin and the superficial temporal fascia) or the plane between the superficial and deep temporal fascia, it will not descend to its desired position beneath the malar eminence and will instead be placed superficial to the zygomatic arch.

The plane deep to the temporalis muscle leads into the infratemporal fossa, which contains the lateral and medial pterygoid muscles, the maxillary artery, the pterygoid venous plexus, the mandibular branch of the trigeminal nerve (V3) and its derivatives, the chorda tympani, and the otic ganglion. Placing the elevating device deep to the temporalis muscle will increase the risk for injury to any of the contents of the infratemporal fossa and will deny the surgeon access to the malar eminence and zygomatic arch. The masseter muscle arises from the lower border and medial surface of the zygomatic arch and extends inferiorly to attach to the lateral aspect of the mandibular ramus. This muscle is not accessible through the temporal scalp.

108
Q

A 28-year-old man is brought to the emergency department one hour after he sustained injuries to the face when he fell down a flight of stairs. Which of the following physical findings indicates that this patient does NOT have an isolated naso-orbital-ethmoid fracture?

(A) Deep nasofrontal angle
(B) Epiphora
(C) Mobility of the maxilla
(D) Telecanthus
(E) Upturning of the nasal tip

A

The correct response is Option C.

In a naso-orbital-ethmoid fracture, which involves the superior nose, canthal-bearing segments, and ethmoid sinuses, the maxilla remains stable because most of it is inferior to the fracture.

A naso-orbital-ethmoid fracture has various physical findings. The depressed nasal segment creates a deep nasofrontal angle. Regional edema can temporarily occlude the puncta, leading to epiphora (tearing). The medial canthi, with or without the small canthal-bearing segments, can be detached, creating telecanthus (an increased distance between the medial canthi). Fracture of the superior half of the nose leads to upturning of the nasal tip.

109
Q

For each patient, select the most appropriate management.

(A) Cranialization of the frontal sinus
(B) Obliteration of the sinus and nasofrontal ducts
(C) Observation for one week
(D) Rigid fixation with preservation of the frontal sinus
(E) Serial CT scans

48 A 22-year-old woman sustains a comminuted fracture of the posterior table of the frontal sinus

49 A 17-year-old boy who sustained a nondisplaced fracture of the frontal sinus has a cerebrospinal fluid leak

50 Five years after sustaining a displaced fracture of the anterior table of the frontal sinus that was not treated operatively, a 43-year-old man has a mucocele

A

The correct response for Item 48 is Option A, for Item 49 is Option C, and for Item 50 is Option B.

In patients who have comminuted fractures of the posterior table of the frontal sinus, rigid fixation cannot be achieved, and management involves cranialization, whereby the sinus is eliminated and thus converted to a portion of the intracranial cavity. This procedure requires neurosurgical exposure.
No operative intervention is required for nondisplaced fractures of the posterior wall of the frontal sinus as long as there is no evidence of cerebrospinal fluid leak. In patients with nondisplaced or minimally displaced fractures of the posterior wall of the frontal sinus who exhibit cerebrospinal fluid leak, the most appropriate initial management is observation for spontaneous resolution for five to seven days. If the cerebrospinal fluid leak continues beyond this point, cranialization of the frontal sinus and repair of the dural laceration are warranted.

Mucocele is a rare, late complication of untreated frontal sinus injury. Symptoms include headache and frontal sinus and orbital pain. Because mucoceles can erode the bony boundaries of the frontal sinus, appropriate management involves exenteration of the mucosa and obliteration of the sinus and nasofrontal ducts.

Serial CT scans are not indicated in the management of fractures of the frontal sinus.

110
Q

A 21-year-old man is brought to the emergency department one hour after sustaining a fracture of the nose during a fistfight. Which of the following physical findings in this patient is most indicative of a concomitant fracture of the septum?

(A) Crepitation
(B) Depression of the nose
(C) Deviation of the nose
(D) Epistaxis
(E) Tearing of the mucosa

A

The correct response is Option E.

In a study of the predictive ability of physical findings in nasal fractures, tearing of the mucosa was the most statistically significant sign of septal fracture. Septal deviation was second, and crepitation was third in the ability to predict septal fracture. Depression of the nose, deviation of the nose, epistaxis, open nasal wounds, and ecchymosis produced low confidence intervals. Thin-section CT scans were not definitive in many cases.

Nasal bone fracture is the most common fracture of the face and the third most common fracture overall. It can result in posttraumatic nasal deformity. The estimated incidence of these secondary deformities ranges from 10% to 50%; many of them result from a septal fracture that was unrecognized and untreated at the time of injury. A septal fracture destabilizes alignment of the nasal bones during healing and increases breathing difficulties. One study using close examination and thin-section CT scans found 50 out of 52 nasal fractures had an associated septal fracture. Another study reported that 47% of nasal fractures had an associated septal fracture. Recognition of septal problems is the single most important step in determining appropriate treatment and attaining excellent outcomes in the primary care of fractures of the nose.
Meticulous septal examination and treatment can reduce the incidence of deformities after fractures of the nose. Septal fractures may be graded according to the severity of the fracture and the amount of displacement. A nasal septum with a fracture and displacement must be relocated into the vomerine groove. The fracture should be directly visualized, if possible, and limited septal repositioning or reconstruction should be performed. Internal stitches and splints may be used to help stabilize the reduction.

111
Q

A 25-year-old woman sustains a frontal sinus fracture in a motor vehicle collision. On physical examination, there is cerebrospinal fluid rhinorrhea. A CT scan of the head shows displacement of both the anterior and posterior walls and fracture lines extending through the nasofrontal ducts. Which of the following is the most appropriate management?

(A) Ablation
(B) Cranialization
(C) Exenteration
(D) Nasalization
(E) Obliteration

A

The correct response is Option B.

Cranialization of the frontal sinus is the most appropriate management of this patient who has a fracture of the posterior table of the frontal sinus with a concomitant cerebrospinal fluid leak. This procedure is often recommended for patients with severe comminution of the posterior table to resolve any cerebrospinal fluid leakage. Bifrontal craniotomy is performed first to repair the dura, and the posterior table of the frontal sinus and associated mucosa are removed. The nasofrontal duct is occluded with a pericranial flap, disrupting the communication of the duct with the frontal sinus. Following surgery, the brain gradually expands to fill the space previously occupied by the frontal sinus.

Ablation of the frontal sinus involves total removal of the anterior and posterior tables. This procedure is no longer performed due to its resultant cosmetic defects.

Exenteration involves removal of the anterior table of the frontal sinus only. Although it results in a cosmetic deformity, it may be considered in patients who have severe damage to the anterior table resulting from infection and who cannot undergo immediate reconstruction.

Nasalization is a technique in which the nasofrontal duct is either stented or enlarged to ensure adequate drainage of the frontal sinus. This procedure is typically used in patients with frontal basilar fractures involving the nasofrontal duct or floor of the sinus.

Obliteration of the frontal sinus is accomplished by removing all of the mucosa within the frontal sinus and allowing the nasofrontal duct to occlude. Graft material can be used for filling, or the space may remain open and eventually close through the process of spontaneous osteogenesis. This technique is recommended for patients in whom the patency of the nasofrontal duct is compromised.

112
Q

A 32-year-old man sustains a fracture of the mandible in a motor vehicle collision. The likelihood of concomitant cervical spine injury in this patient is closest to

(A) 10%
(B) 20%
(C) 30%
(D) 40%
(E) 50%

A

The correct response is Option A.

In patients who sustain facial fractures in motor vehicle collisions, the incidence of cervical spine injury has been shown to range from 5% to 15%, according to the results of multiple studies. Overall, multiple studies have reported the incidence of concomitant injuries associated with facial fractures sustained during motor vehicle collisions as ranging from 11% to 99%. These injuries are most likely to include closed head injuries, soft-tissue lacerations to the face, head, or other regions, and fractures of the ribs, pelvis, and lower extremities.

Because of the correlation between facial fractures and cervical spine injuries, standard Advanced Trauma Life Support (ATLS) protocols recommend that the cervical spine be immobilized until the absence of cervical spine injury can be documented definitively. These injuries can result in paresis, paraplegia, and even death; therefore, the surgeon must maintain a high index of suspicion in any patient who sustains a mandibular fracture in a motor vehicle collision. Even patients who have mandibular fractures resulting from physical altercations should be evaluated carefully, although the incidence of concomitant cervical spine injury is not as high as in those patients who are involved in motor vehicle collisions.

Patients with mandibular fractures often have other associated injuries, such as lacerations of the face and head and other associated facial fractures. These patients should also be evaluated for potential closed head injury, which is a life-threatening concern associated with high mortality rates.

113
Q

A 25-year-old man has diplopia two days after sustaining an orbital fracture in a motor vehicle collision. On physical examination, he does not have enophthalmos; review of CT scans obtained immediately after injury shows no bony displacement or entrapment of the orbital contents within the fracture.

Which of the following is the most appropriate next step in management?

(A) Observation
(B) Administration of a corticosteroid
(C) Administration of a diuretic
(D) MRI
(E) Immediate operative exploration

A

The correct response is Option A.

Conservative management is recommended in this patient who has diplopia after sustaining an undisplaced, stable orbital fracture without entrapment or enophthalmos. Diplopia often occurs following orbital trauma and may be caused by edema, neurovascular or muscle injury, or entrapment of surrounding structures within the fracture. If entrapment is the cause, surgical release should be performed within the first 24 to 48 hours after injury to prevent permanent muscle damage. However, if there is no entrapment, as in this patient, observation for 10 to 14 days is most appropriate. This will allow for resolution of any edema or temporary palsy. Repeat evaluation can then be performed at this time.

CT scan, not MRI, is most appropriate for evaluating the condition of the fracture site.

Corticosteroids or diuretics are not indicated in patients who have diplopia following orbital fracture.

Operative exploration is appropriate for reduction of bony fragments, release of entrapped, compressed, or prolapsed tissues, and accurate restoration of orbital volume, but is not the most appropriate next step in this patient.

114
Q

A 16-year-old basketball player is undergoing evaluation 12 hours after sustaining a nasal fracture in a basketball game. Physical examination of the fracture site shows marked edema. The radix is stable. A single fracture fragment is displaced from the bony nasal pyramid. Intranasal examination shows septal hematomas bilaterally.

Which of the following is the most appropriate management?

(A) Drainage of the septal hematomas followed by closed reduction and splinting in three days
(B) Immediate operative drainage of the septal hematomas, followed by closed reduction and splinting
(C) Drainage of the septal hematomas and intraoral open reduction and internal fixation of the nasomaxillary suture line in three days
(D) Closed reduction of the nasal fracture in three days, followed by submucosal resection and inferior turbinate infracture in six months
(E) CT scan of the face to rule out a naso-orbitoethmoid fracture, followed by open reduction and internal fixation of the fractures and septoplasty

A

The correct response is Option A.

In this patient who has sustained a nasal fracture with displacement of one fragment, intranasal examination shows septal hematomas bilaterally. Therefore, the most appropriate management is drainage of the hematomas, followed by closed reduction and splinting in three days. It is imperative to drain the hematomas immediately to prevent the development of complications, including thickening of the septum (ie, “cauliflower” deformity) or dissolution and collapse of the septum, which will ultimately result in a saddle-nose deformity. Because this patient has significant swelling, closed reduction should be delayed. After the swelling has decreased (typically at three to five days after injury), the septum and nasal pyramid should be reduced, and the nasal pyramid should then be splinted.

Immediate closed reduction is difficult in any patient with significant edema.

Internal fixation and/or septoplasty are not required for management of uncomplicated nasal fractures.

Submucous resection and turbinectomy are rarely necessary because most patients with nasal fractures do not experience airway compromise.

115
Q

A 23-year-old man is undergoing evaluation one week after sustaining a nasal fracture. Each of the following is appropriate management of this patient’s injuries EXCEPT

(A) closed realignment of the nasal fracture with forceps
(B) drainage of septal hematomas
(C) intranasal packing
(D) osteotomy and realignment of the nasal fracture
(E) use of a dorsal nasal splint

A

The correct response is Option D.

Complications associated with fracture of the nasal bones include hemorrhage and hematoma. Bleeding is common with nasal trauma because of the rich blood supply of the mucoperichondrium. Fracture of the nasal septum can lead to hematoma, which frequently occurs bilaterally, as septal fractures communicate between both sides of the nose. If untreated, a septal hematoma can become thick and fibrotic, obstructing the nasal passage, or can cause pressure necrosis of the nasal mucosa and cartilage, ultimately leading to septal perforation. Therefore, appropriate management involves incision along the base or most inferior portion of the hematoma, which will allow for drainage and prevent blood from refilling the cavity. In addition, closed reduction is appropriate for septal fractures and deviated nasal bones. Intranasal packing and dorsal nasal splints are typically used to aid in maintaining the reduction.

Osteotomy should not be performed in patients with acute fractures because nasal collapse may result. This procedure should be delayed until the fracture has healed significantly.

116
Q

A 38-year-old man sustains panfacial fractures in a motor vehicle collision. On physical examination, the midface is unstable. Radiographs show bilateral displaced fractures of the condylar neck. Which of the following of the most appropriate management?

(A) Open reduction and internal fixation of the midface fractures followed by a soft diet for four weeks
(B) Open reduction and internal fixation of the midface fractures followed by placement of a mandibular external fixator
(C) Open reduction and internal fixation of the midface fractures followed by intermaxillary fixation of the mandible
(D) Open reduction and internal fixation of the condylar neck fractures followed by open reduction and internal fixation of the midface fractures
(E) Open reduction and internal fixation of the midface fractures followed by open reduction and internal fixation of the condylar neck fractures

A

The correct response is Option D.

In this patient who has fractures of the midface and condylar neck, the most appropriate management is open reduction and internal fixation of the condylar fractures, followed by open reduction and internal fixation of the midface fractures. Reducing the mandible first is the only method for reestablishing the appropriate height of the posterior face; the midface can then be repaired using the mandible as a reference.

Conservative measures, such as initiation of a soft diet without fixation of the mandible, are not appropriate in patients with fractures of the condylar neck. External fixators are typically applied in patients who have comminuted fractures of the mandible, or when early open reduction and internal fixation are not possible, such as in patients who are medically unstable and cannot undergo surgical procedures.

117
Q

A 24-year-old woman undergoes Le Fort I osteotomy with maxillary impaction and bilateral sagittal split osteotomy with mandibular advancement. Following release of intermaxillary fixation six weeks later, the patient has an anterior open bite. Which of the following is the most likely cause of this finding?

(A) Improper intraoperative seating of the condyles in the glenoid fossae
(B) Improper presurgical orthodontic treatment
(C) Loosening of all plates of the rigid internal fixation
(D) Parafunctional habits, such as tongue thrusting
(E) Progressive resorption of the condyles

A

The correct response is Option A.

This patient’s anterior open bite, seen six weeks after surgery, is most likely a result of improper intraoperative seating of the condyles in the glenoid fossae. Once the osteotomy has been completed, it is important to release the patient from intemaxillary fixation in order to ensure that the condyles are properly seated within the fossae. The occlusion and path of the opening of the mandible are examined at this time.

Improper presurgical orthodontic treatment would result in a late recurrence of malocclusion. It is unlikely that all of the plates of the rigid internal fixation would loosen over the six-week fixation period. Parafunctional habits, such as tongue thrusting, are a late cause of anterior open bite. Progressive condylar resorption, which manifests as condylar shortening, decreased height of the posterior face, and clockwise rotation of the mandible, is a late cause of open bite occurring primarily in young women. The cause of this condition is unknown.

118
Q

A 24-year-old man is brought to the emergency department after being struck in the face. CT scan of the face shows an orbital blow-out fracture. Which of the following findings is an indication for operative intervention?

(A) Blood in the maxillary sinus
(B) Diplopia on primary gaze
(C) Hypesthesia in the infraorbital nerve distribution
(D) Orbital floor defect greater than 2 cm
(E) Subconjunctival hematoma

A

The correct response is Option D.

In a patient who has sustained an orbital blow-out fracture, indications for surgical exploration include an orbital floor defect of greater than 2 cm, abnormally low vertical height of the globe, and the presence of other fractures. Operative exploration should be performed in patients who have symptomatic diplopia in association with positive findings on forced duction testing. Patients who have symptoms of extraocular muscle entrapment that do not resolve in one week or indications of muscle entrapment on radiographs obtained one week after surgery should undergo additional exploration.

Diplopia on primary gaze typically improves within the first two weeks after surgery. The presence of blood in the maxillary sinus and hypesthesia in the distribution of the infraorbital nerve is common in patients with minor orbital fractures, and surgical intervention is not required. Likewise, subconjunctival hematoma is not an indication for operative intervention

119
Q

An 18-year-old man has a displaced fracture of the anterior table of the frontal sinus. Which of the following statements is most accurate regarding glue fixation of this fracture using butyl-2-cyanoacrylate?

(A) Facial bone healing will be partially impeded
(B) Glue fixation is more time consuming than plate and screw fixation
(C) Glue fixation will provide compressive forces comparable to either resorbable or titanium plate and screw fixation
(D) The glue will not adhere to a moist bone surface
(E) Inflammation resulting from breakdown products of butyl-2-cyanoacrylate will lead to scarring and possible brain damage

A

The correct response is Option C.

According to the results of in vitro studies, plate and screw fixation devices have been shown to tolerate higher distraction forces than glue fixation. Plates and screws also provide greater biomechanical stability than butyl-2-cyanoacrylate in bones, such as the mandible, that absorb large forces. However, in the thin bone fragments of the anterior table of the frontal sinus, which are affected by small compressive forces, butyl-2-cyanoacrylate has been shown to provide fixation stability that is comparable to either resorbable or titanium plate and screw fixation. In addition, fixation of the thin bone fragments of this region is limited by the pull-out strength of the screws to the applied bone segments, which is not significantly greater than the adhesive strength of cyanoacrylate to bone.

Facial bone healing is not impeded by cyanoacrylates; instead, according to the results of some studies, healing is believed to be augmented.

Use of tissue adhesives requires less operative time than fixation with plates and screws. Cyanoacrylates set within seconds, and the need to contour plates accurately is eliminated.

Bone surfaces are often moist intraoperatively, and cyanoacrylates will adhere to these moist surfaces.

Inflammation is rare in the tissues adjacent to those exposed to butyl-2-cyanoacrylate or its breakdown products; harmful side effects are minimal. Damage to the underlying brain was not shown to occur in animal studies in which cyanoacrylate was used for cranial fixation.

120
Q

In a patient who has sustained a fracture of the zygomaticomaxillary complex, which of the following anatomic structures is most useful for reduction of the fracture components?

(A) Inferior orbital rim
(B) Lateral orbital wall
(C) Orbital floor
(D) Zygomatic arch
(E) Zygomaticomaxillary plane

A

The correct response is Option B.

In a patient who has sustained a fracture of the zygomaticomaxillary complex, the lateral orbital wall and the sphenoid wing can be used as landmarks to obtain the most accurate reduction. Visualization of these structures from inside the orbit will allow for visualization of the relatively flat plane of the orbital portion of the zygoma and the relatively flat portion of the sphenoid wing. An appropriate anatomic reduction is attained when these two areas are smoothly aligned.

Although the inferior orbital rim, orbital floor, zygomatic arch, and zygomaticomaxillary plane can be useful landmarks for individual alignment, use of any of these sites for reduction of the zygomaticomaxillary complex will result in significant rotational malalignment at the other fracture sites.

121
Q

Which of the following structures is incised when a preseptal transconjunctival incision is performed in patients with fractures of the orbital floor?

(A) Capsulopalpebral fascia
(B) Levator palpebrum
(C) Orbicularis oculi muscle
(D) Orbital septum
(E) Tarsus

A

The correct response is Option A.

The preseptal transconjunctival incision has been used with increasing frequency in recent years because any associated scars are better concealed and the risk for eyelid retraction is lower than with transcutaneous approaches. With this technique, the incision is made through the conjunctiva below the tarsus of the lower eyelid. The capsulopalpebral fascia (retractors of the lower eyelid) is incised and the plane between the orbicularis oculi muscle and the septum is entered. The periosteum of the orbital rim is then incised to expose the fracture.

The levator palpebrum, an elevator of the upper eyelid, is not involved in repair of orbital floor fractures.

122
Q

A patient has dilation of the right pupil immediately after undergoing open reduction and internal fixation of an orbitozygomatic fracture on the right. On examination, the right pupil is unresponsive to direct light stimulation, and there is no consensual response to light.

These findings are best explained by injury to which of the following structures?

(A) Globe
(B) Optic nerve
(C) Ocular parasympathetic nerves
(D) Ocular sympathetic nerves
(E) Trochlear nerve

A

The correct response is Option C.

Complete inability to constrict the right pupil associated with absence of direct and consensual responses to light is most likely caused by compromised function of the ocular parasympathetic innervation. Because the parasympathetic fibers travel with the oculomotor nerve and inferior oblique muscle, they can be injured during reduction or fixation of fractures in the region of the orbit and zygoma, especially with manipulation of the muscle. Use of topical mydriatic agents, such as epinephrine, can also result in these findings.

Fixed dilation of the pupil would not occur in a patient who sustained injury to the globe. Trauma to the optic nerve would result in a relative afferent pupillary defect, in which the affected eye cannot perceive light. In patients with this finding, known as a Marcus-Gunn pupil, direct response to light is impaired, but consensual response is preserved. Injury to the ocular sympathetic nerves would disrupt the dilatory reflex of the pupil, while injury to the trochlear (IV) nerve would affect the actions of the superior oblique muscle.

123
Q

Patients with displaced zygomatic fractures are most likely to have which of the following findings at the palpebral fissure?

(A) Anterior displacement
(B) Downward cant
(C) Posterior displacement
(D) Rounding
(E) Upward tilt

A

The correct response is Option B.

Fractures of the orbitozygomatic complex most commonly have break lines through the midinfraorbital rim that extend into the orbital floor and lateral orbital wall along the zygomaticosphenoid suture and end toward the zygomaticofrontal suture. Complete fractures have break lines that extend from the infraorbital rim through the zygomaticomaxillary buttress inferiorly and zygomatic arch laterally. Fractures of the orbitozygomatic complex can be displaced en bloc or rotationally; most displaced zygomatic fractures are depressed and rotated laterally. In patients with displaced, laterally rotated fractures of the orbitozygomatic complex, the lateral canthus attached to Whitnall’s tubercle pulls the palpebral fissure into a downward cant. This finding is also seen in patients with medially rotated fractures because the unopposed pull of the attached masseter muscle contributes to downward movement of the zygoma. Lateral displacement of the lateral canthal ligament is also associated with this downward movement, leading to an increase in the width of the fissure.

There is no substantial anterior or posterior displacement of the palpebral fissures with fractures of the orbitozygomatic complex.

Rounding of the palpebral fissure can occur with the less common en bloc type of medial displacement but is more frequent with detachment of the lateral canthal ligament or with a severely comminuted fracture of the frontal process of the zygoma.

The fracture patterns associated with orbitozygomatic complex fractures would not result in upward displacement of Whitnall’s tubercle.

124
Q

A 35-year-old man has persistent enophthalmos 18 months after undergoing open reduction of a fracture of the orbital floor and zygoma. Forced duction testing shows no restriction of eye motion. Which of the following is the most likely cause of this patient’s enophthalmos?

(A) Fat atrophy
(B) Fibrosis of the extraocular muscles
(C) Herniated contents of the orbit within the maxillary sinus
(D) Inadequate fracture reduction
(E) Scar contracture

A

The correct response is Option D.

Persistent enophthalmos following facial trauma is primarily caused by increased bony orbital volume secondary to inadequate fracture reduction. An appropriate anatomic reduction should be the initial goal of surgery; bone grafting may be required to restore orbital volume to normal levels.

Fat atrophy, extraocular muscle fibrosis, herniation of orbital contents, and scar contracture can also contribute to the discrepancy between traumatic and nontraumatic orbital volume, but are less likely to contribute to postoperative enophthalmos than inadequate fracture reduction. In addition, extraocular muscle fibrosis and/or scarring would typically be associated with limited motion of the globe on forced duction testing.

125
Q

A 25-year-old man sustains a fracture of the frontal sinus in a motor vehicle collision. A CT scan of the frontal sinus shows a comminuted fracture of the anterior table and a linear nondisplaced fracture of the posterior table. There is no evidence of cerebrospinal fluid leak. Following removal of the anterior table fragments during surgical exploration, methylene blue is instilled into the sinus and passes into the nasal cavity.

Which of the following is the most appropriate management?

(A) Fixation of the anterior table bone fragments only
(B) Enlargement of the nasofrontal duct
(C) Ablation of the frontal sinus
(D) Cranialization of the frontal sinus
(E) Obliteration of the frontal sinus

A

The correct response is Option A.

In this patient who has sustained a frontal sinus fracture, the most appropriate management is fixation of the anterior table fragments. Intraoperative testing showing unobstructed passage of methylene blue into the nasal cavity indicates an intact sinus that should remain functional postoperatively.

Enlargement of the nasofrontal duct with stent placement is a new procedure that has not yet gained wide acceptance and is only indicated for patients with compromise of the nasofrontal duct. Ablation of the frontal sinus involves removal of the anterior table, which allows the forehead skin to collapse into the sinus. This procedure is no longer used because of its resultant cosmetic defects.
With cranialization of the frontal sinus, the posterior wall is removed and the intracranial contents (dura and brain) gradually expand anteriorly to fill the open space within the cranium. This procedure is often recommended for patients with severe comminution of the posterior table of the frontal sinus, particularly in the presence of a cerebrospinal fluid leak. Frontal sinus obliteration is appropriate for correction of nasofrontal duct obstruction, which in patients with frontal sinus fractures can be confirmed by failure of the methylene blue to pass into the nasal cavity.

126
Q

A 25-year-old woman is brought to the emergency department after sustaining injuries in a motor vehicle collision. The patient is alert on initial evaluation and has a Glasgow Coma Scale score of 15. On physical examination, there is periorbital ecchymosis on the right, loss of sensation in the area of the left forehead, ptosis of the right upper eyelid, right-sided ophthalmoplegia, and a fixed, dilated pupil. Consensual light reflex is intact.

These findings are most consistent with which of the following?

(A) Dehiscence of the levator palpebrae superioris muscle
(B) Entrapment of the inferior rectus muscle
(C) Orbital apex syndrome
(D) Retrobulbar hematoma
(E) Superior orbital fissure syndrome

A

The correct response is Option E.

This patient has findings most consistent with superior orbital fissure syndrome, a high-velocity injury caused by extension of an orbital fracture into the superior orbital fissure. The oculomotor (III), trochlear (IV), and abducens (VI) nerves, and the ophthalmic division of the trigeminal nerve (V1) pass through the superior orbital fissure, which is formed from the greater and lesser wings of the sphenoid bone. Affected patients typically have paralysis of the extraocular muscles and the levator palpebrae superioris muscle resulting from injury to multiple nerves. If the ophthalmic division of the trigeminal nerve is involved, the patient will have anesthesia of the forehead, eyebrow, and upper eyelid.

The other conditions listed above would result in some but not all of the findings seen in this patient. Dehiscence of the levator palpebrae superioris muscle manifests as isolated ptosis of the eyelid. In patients with entrapment of the inferior rectus muscle, complete ophthalmoplegia would not be seen. Orbital apex syndrome is characterized by the findings associated with superior orbital fissure syndrome as well as blindness resulting from involvement of the optic nerve. Patients with retrobulbar hematomas may also experience the onset of blindness caused by increased intraocular pressure and proptosis.

127
Q

During the application of rigid fixation in a 9-year-old child who has sustained a Le Fort I fracture, which of the following permanent tooth buds is at greatest risk for injury?

(A) Canine
(B) Central incisor
(C) First molar
(D) First premolar
(E) Lateral incisor

A

The correct response is Option A.

A 9-year-old child has mixed dentition; deciduous (primary) and permanent (secondary) teeth are present within the oral cavity simultaneously. This commonly occurs between ages 8 and 10 years. As a result, it is imperative for the surgeon to be aware of the potential for injury to the tooth buds when applying rigid fixation for management of pediatric maxillofacial fractures. The permanent canine teeth, or cuspids, erupt between ages 10 and 11 years. Therefore, the tooth buds can be injured during the application of rigid fixation in the region of the nasomaxillary buttress.

The central and lateral incisors erupt between ages 6 and 8 years. The permanent first molars erupt between ages 6 and 7 years, and the first premolars erupt between ages 8 and 9 years. Therefore, in a 9-year-old child, all of these teeth should have already begun erupting into the oral cavity, and the risk for injury to the tooth bud will be minimal.

128
Q

A 24-year-old man sustains a Le Fort I fracture on the left and a Le Fort III fracture on the right in a motor vehicle collision. In this patient, which of the following bones is most likely to be fractured on both sides of the face?

(A) Ethmoid
(B) Orbital floor
(C) Palate
(D) Pterygoid plate
(E) Zygoma

A

The correct response is Option D.

In patients with Le Fort I fractures, the pterygoid plate is most likely to be affected, regardless of the type of fracture, because it lies posterior and thus forms the most posterior aspect of the fracture. The ethmoid, orbital floor, palate, and zygoma are each involved in some but not all types of Le Fort I fractures; an illustration is shown above.

129
Q

Which of the following is a late complication following frontal sinus fracture?

(A) Cerebrospinal fluid leak
(B) Epistaxis
(C) Meningitis
(D) Mucocele
(E) Sinusitis

A

The correct response is Option D.

Patients with frontal sinus fractures, especially those involving the posterior table, can develop complications secondary to nasofrontal duct obstruction, dural injury, or entrapment of the sinus mucosa. Acute complications within the first few hours after injury can include epistaxis, cerebrospinal fluid leak, meningitis, and intracranial injury. The most common subacute complications occurring within the first few weeks following fracture are frontal sinusitis, mucocele, and meningitis. Long-term complications, such as osteomyelitis, mucocele, and chronic intracranial or orbital abscesses, can occur as late as several years after injury. Cosmetic deformities may also be seen late.

Because most complications occur in patients with fractures of the posterior table, appropriate reduction of all posterior fragments and repair of all dural tears are recommended. Reconstruction with pericranial flaps can decrease the risk for development of complications by distancing the brain tissue from the fracture site. Cranialization, which involves removal of mucosa and plugging of the nasofrontal ducts, can also be used to minimize complications. In patients who have nonfunctioning nasofrontal ducts, obliteration of the frontal sinus may be considered.

130
Q

In a 32-year-old man who sustained a panfacial fracture in a high-speed motor vehicle collision, what is the approximate risk for concomitant cervical spine injury?

(A) 5%
(B) 10%
(C) 15%
(D) 20%
(E) 25%

A

The correct response is Option B.

Patients who are involved in high-speed motor vehicle collisions have a risk for concomitant cervical spine injury that is closest to 10%. The incidence of associated cervical spine injury is greatly increased in these patients. Therefore, a patient who has sustained a facial fracture in a high-speed collision should be evaluated for a potential cervical spine injury prior to treatment of the facial fracture.

131
Q

In a patient who has sustained a fracture of the zygomaticomaxillary complex (tripod fracture), accurate reduction of the fracture components is most likely to be accomplished with the use of which of the following anatomic structures?

(A) Inferior orbital rim
(B) Lateral orbital wall
(C) Orbital floor
(D) Zygomatic arch
(E) Zygomaticomaxillary plane

A

The correct response is Option B.

Appropriate reduction of a fracture of the zygomaticomaxillary complex involves a three-dimensional process. In order to obtain the most accurate reduction, the lateral orbital wall and sphenoid wing should be visualized from inside the orbit. This will allow for visualization of the relatively flat plane of the orbital portion of the zygoma and the relatively flat portion of the sphenoid wing; accurate reduction is obtained when these two areas are aligned completely.

Although the inferior orbital rim, orbital floor, zygomatic arch, and zygomaticomaxillary plane are helpful sites for alignment individually, use of any of the sites as a landmark for accurate fracture reduction can result in significant rotational malalignment at the other sites.

132
Q

Which of the following fixation materials causes the LEAST amount of scatter on CT scan?

(A) Polylactic acid
(B) Stainless steel
(C) Tantalum
(D) Titanium
(E) Vitallium

A

The correct response is Option A.

The copolymer of polylactic acid and L-glycolic acid (Lactasorb) is a nonmetallic substance that is currently used in craniomaxillofacial fixation, is not visible on plain radiographs and/or CT scans, and is completely resorbed within one year following implantation.

Among metals used in fixation, stainless steel alloy (comprised of chromium, nickel, and molybdenum) exhibits the most scatter, while titanium and Vitallium (cobalt-chromium alloy) produce the least scatter. Tantalum is not currently used for craniomaxillofacial fixation because it exhibits inadequate mechanical properties.

133
Q

Which of the following is the most common cause of posttraumatic enophthalmos?

(A) Fat atrophy
(B) Increased volume of the bony orbit
(C) Ligament disruption
(D) Orbital roof defect
(E) Soft-tissue contracture

A

The correct response is Option B.

Enophthalmos, or posterior displacement of the globe within the bony orbit, is most frequently caused by an increase in bony orbital volume. Because the positioning of the globe is determined by the bony orbit, ligamentous system, and orbital fat, changes in any of these contributing factors occurring as a result of trauma can lead to a change in globe position. However, volumetric studies have pointed to enlargement of orbital volume as the predominant cause of enophthalmos. As such, surgical correction of facial fractures should always include the confirmation and/or re-establishment of normal bony orbital volume. This involves confirmation of the correct positioning of the medial and lateral orbital floors.

Disruption of the orbital ligaments can also result in posttraumatic enophthalmos, especially in relation to increased bony orbital volume. In certain patients who have increased orbital volume, intact ligaments can function to maintain the globe in its normal position. However, if the ligaments are disrupted, the soft tissues can contract, resulting in a change in both orbital shape and volume. If this is not corrected, the globe will be positioned abnormally following healing, and the enophthalmos will then be difficult to correct.

Defects in the orbital roof are not commonly cited as a cause of enophthalmos because superior displacement of the globe into the anterior cranial fossa is rare.

134
Q

In a patient undergoing surgical management of a Le Fort I fracture, rigid fixation is applied using metal plates and screws. When maxillomandibular fixation is removed to confirm the occlusal relationship, a unilateral posterior open bite is noted.

Which of the following is the most appropriate next step in management?

(A) Fixation with elastic banding on the side of the open bite for four weeks
(B) Re-establishment of maxillomandibular fixation for six weeks
(C) Removal of all rigid fixation and disimpaction of the maxillary fracture
(D) Removal of all rigid fixation followed by wire fixation of the fracture sites
(E) Replacement of the metal plates with absorbable (Lactasorb) plates on the side of the open bite

A

The correct response is Option C.

The most important management principal in this patient who has a Le Fort I fracture involves re-establishing the pretraumatic maxillomandibular orientation. Erich arch bars can be used to re-establish occlusion in patients with
If the maxilla has been impacted into the pterygoid plates, the occlusion will be angled superiorly toward the side of impaction. Therefore, if rigid fixation is applied before the fracture site is disimpacted, the patient will have an open bite on removal of maxillomandibular fixation. In order to prevent this, the impacted segments should be mobilized prior to the application of rigid maxillomandibular fixation, and an even plane should be established to correct the open bite.

Wires, elastic bands, and absorbable plates (Lactasorb) can be used to re-establish maxillomandibular fixation but will not correct the open bite.

135
Q

A 27-year-old woman has numbness of the left cheek after being hit in the eye with a tennis ball. Radiographs show an orbital blowout fracture. Which of the following is the most likely cause of the numbness?

(A) Edema of the skin over the cheek
(B) Entrapment of the infraorbital nerve distal to the foramen
(C) Fracture of the body of the zygoma
(D) Fracture of the infraorbital rim
(E) Injury of the infraorbital nerve within the orbital floor

A

The correct response is Option E.

A patient who experiences paresthesias of the cheek skin after sustaining a pure blowout fracture of the orbital floor usually has an injury of the infraorbital nerve. This nerve, which is a branch of the maxillary division of the trigeminal nerve (V2), courses within the inferior orbital canal along the floor of the orbit and exits the body of the zygoma through the infraorbital foramen.

Patients with pure orbital blowout fractures rarely have involvement or fracture of the infraorbital rim or body of the zygoma. The fracture fragments from the orbital floor and medial orbital wall are typically displaced into the sinus. Edema usually occurs in the periorbital region and not the soft tissues of the cheek. Although there are no sites to entrap the nerve distally, patients with more extensive periorbital fractures can have an injury of the infraorbital nerve, but will exhibit additional physical findings.

136
Q

A 30-year-old man sustains an injury to the left side of the face in a fistfight. Radiographs show an isolated fracture of the zygomatic arch; surgical reduction of the fracture is planned. Following incision in the temporal region, the instrument should be passed immediately beneath which of the following layers of the scalp?

(A) Hair follicles
(B) Subcutaneous fat of the scalp
(C) Superficial temporal fascia
(D) Deep temporal fascia
(E) Temporalis muscle

A

The correct response is Option D.

In the temporal (Gillies) approach, a technique used for operative fixation of zygomatic fractures, the initial scalp incision extends down through skin, subcutaneous tissue, and the superficial and deep temporal fascia. An elevating device is then inserted to raise the fractured arch in order to facilitate effective reduction. This instrument is passed immediately beneath the deep layer of the deep temporal fascia, which lies just beneath the arch. Guiding the instrument beneath the deep layer of the deep temporal fascia allows for careful elevation of the fractured bone.

Because the hair follicles, subcutaneous fat, and superficial temporal fascia are all positioned superficial to the zygomatic arch, passing the elevating device beneath any of these structures will not allow for elevation of the zygoma and may instead result in damage to the frontal branch of the facial nerve. Guiding the elevating device beneath the temporalis muscle is a less optimal approach; if the surgeon passes the instrument too deeply, it will also pass beneath the coronoid process and make elevation of the zygomatic arch more difficult.

137
Q

In a 29-year-old woman who sustained trauma to the face during a rugby game four weeks ago, intranasal inspection with a nasal speculum shows a perforation of the nasal septum. A physical examination and radiographs obtained in the emergency department at the time of initial injury showed findings consistent with a displaced nasal fracture.

Which of the following is the most likely cause of the septal deformity?

(A) Foreign body perforation
(B) Nasal bone puncture
(C) Septal hemangioma
(D) Septal hematoma
(E) Turbinate bone perforation

A

The correct response is Option D.

The nasal septal perforation seen in this patient is most likely caused by a septal hematoma that was not diagnosed at the time of injury. Because undiagnosed septal hematomas can cause pressure necrosis of the nasal mucosa and cartilage and ultimately lead to septal perforation, intranasal inspection using a nasal speculum should be performed in any patient who has sustained a nasal fracture and/or trauma to the nasal bones. Any hematoma seen on intranasal inspection should then be immediately drained. A #11 blade or a #16-18 needle and syringe can be used for drainage of the hematoma in order to relieve pressure and allow blood to flow into the nose.

A foreign body, nasal bone puncture, septal hemangioma, or turbinate bone perforation is unlikely to result in a perforation of the nasal septum.

138
Q

A 24-year-old man has moderate ectropion three weeks after undergoing open reduction and internal fixation of a malar complex fracture through subciliary and intraoral incisions. Operative exploration of the orbital floor was performed to confirm fracture reduction.

Which of the following anatomic sites is the most likely origin of this patient’s ectropion?

(A) Lateral canthal ligament
(B) Orbicularis oculi
(C) Orbital septum
(D) Skin
(E) Tarsus

A

The correct response is Option C.

This patient has ectropion that has most likely been caused by edema and scar contracture of the orbital septum. Incision into the first eyelid crease is recommended to prevent the development of this complication. This incision has the lowest associated incidence of ectropion because it preserves the innervation of the pretarsal portion of the orbicularis oculi; therefore, normal eyelid tone is maintained. Although there is no true skin shortage seen in this patient, longstanding ectropion can lead to further untoward complications.

The lateral canthal ligament and tarsus would not be affected by this incisional approach. Cutaneous deficiencies should not occur because skin should not be resected.

139
Q

A 35-year-old woman sustains blunt trauma to the preauricular region during a field hockey game. Radiographs show a fracture of the condylar neck with medial displacement of the proximal fracture fragment. Which of the following muscles most likely contributes to the fracture displacement?

(A) Buccinator
(B) Lateral pterygoid
(C) Masseter
(D) Medial pterygoid
(E) Temporalis

A

The correct response is Option B.

In this patient who has a fracture of the condylar neck, displacement of the proximal fracture fragment is most likely caused by the action of the lateral pterygoid muscle. This muscle, which has two heads, is the only muscle that inserts directly on the mandibular condyle, on its anterior portion. The inferior head of the lateral pterygoid muscle arises from the lateral pterygoid plate and inserts into the anterior surface of the neck of the condyle; it acts to open the mandible. The superior head arises from the infratemporal crest, infratemporal surface of the greater wing of the sphenoid bone, and a portion of the squamous part of the temporal bone and inserts into the capsule and articular disk of the TMJ; it contributes to the motion of the articular disk. In patients with condylar fractures, the unopposed force of the lateral pterygoid muscle pulls the condylar head medially.

The buccinator muscle lines the oral cavity and has no attachments within the TMJ. The masseter muscle has its bony attachments at the zygomatic arch and along the inferior border of the lateral portion of the mandibular angle and body. The medial pterygoid muscle arises within the pterygoid fossa and inserts into the medial surface of the mandibular angle and ramus. The temporalis muscle originates at the insertion of the infratemporal fossa of the temporal bone and the coronoid process. This muscle does not attach to the TMJ.

140
Q

A 45-year-old man sustains an isolated fracture of the body of the zygoma that is displaced inferiorly and posteriorly. The accurate alignment of which of the following anatomic structures provides the most useful guide for surgical reduction of the fracture?

(A) Anterior maxillary wall
(B) Frontozygomatic suture
(C) Inferior orbital rim
(D) Lateral orbital wall
(E) Zygomatic arch

A

The correct response is Option D.

In this 51-year-old man who has a significantly displaced fracture of the body of the zygoma, accurate fracture reduction can be best accomplished using the lateral orbital rim as a guide. Although each of the structures listed above will provide useful information for accurate reduction of this type of fracture, the lateral orbital rim is least likely to be comminuted, and reduction of the lateral orbital wall with the greater wing of the sphenoid bone will allow for simultaneous correction of the reduction in all planes. Use of the lateral orbital rim for accurate fracture reduction will reduce the patient’s risk for development of malar flattening, enophthalmos, and other secondary sequelae.

Reduction of the frontozygomatic suture will correct the inferior displacement but may leave persistent rotational changes. Likewise, although reduction of the anterior maxillary wall, the inferior orbital rim, or the zygomatic arch alone without reduction of adjacent structures will allow correction in one plane, the rotational defects may remain undetected.

141
Q

Zygomatic osteotomies for increased anterior projection of the cheek bone will be most beneficial for which of the following patients?

(A) A 15-year-old boy who has Treacher Collins syndrome
(B) A 23-year-old woman who desires primary cheek augmentation
(C) A 32-year-old man who has facial flattening after sustaining panfacial fractures
(D) A 45-year-old woman who is undergoing rhytidectomy and bicoronal forehead lifting
(E) A 50-year-old man who has loss of malar projection one week after sustaining a zygomatico-orbital fracture

A

The correct response is Option C.

The 32-year-old man with panfacial fractures is the sole candidate of those listed above who should undergo zygomatic osteotomies, which are typically indicated to provide increased malar projection in patients who have scarring or inadequate vascularization of the soft tissues. Panfacial fractures are frequently caused by high velocity injury to the facial skeleton, often resulting in an equal distribution of force to all of the tissues of the face. Scarring and disturbance of soft-tissue vascularization are frequent. Although zygomatic osteotomies are never indicated in patients who have acute injuries, they can be considered if facial deformities persist following fracture repair.

In contrast, alloplastic augmentation is the procedure of choice in all other patients who require increased projection; this technique is easier to perform and is associated with fewer complications. The implants are easier to form and mold for the individual face.

In the 50-year-old man with inadequate malar projection following zygomatico-orbital fracture, open reduction and internal fixation are most appropriate.